21.07.2014 Views

CHAPTER FIFTEEN

CHAPTER FIFTEEN

CHAPTER FIFTEEN

SHOW MORE
SHOW LESS

Create successful ePaper yourself

Turn your PDF publications into a flip-book with our unique Google optimized e-Paper software.

<strong>CHAPTER</strong> <strong>FIFTEEN</strong><br />

15.1 SOLUTIONS 1039<br />

Solutions for Section 15.1<br />

Exercises<br />

1. The point A is not a critical point and the contour lines look like parallel lines. The point B is a critical point and is a local<br />

maximum; the point C is a saddle point.<br />

2. To find the critical points, we solve f x = 0 and f y = 0 for x and y. Solving<br />

f x = 2x − 2y = 0,<br />

f y = −2x + 6y − 8 = 0.<br />

We see from the first equation that x = y. Substituting this into the second equation shows that y = 2. The only critical<br />

point is (2, 2).<br />

We have<br />

D = (f xx)(f yy) − (f xy) 2 = (2)(6) − (−2) 2 = 8.<br />

Since D > 0 and f xx = 2 > 0, the function f has a local minimum at the point (2, 2).<br />

3. The partial derivatives are<br />

Set f x = 0 and f y = 0 to find the critical point, thus<br />

f x = −6x − 4 + 2y and f y = 2x − 10y + 48.<br />

2y − 6x = 4 and 10y − 2x = 48.<br />

Now, solve these equations simultaneously to obtain x = 1 and y = 5.<br />

Since f xx = −6, f yy = −10 and f xy = 2 for all (x, y), at (1, 5) the discriminant D = (−6)(−10) − (2) 2 = 56 ><br />

0 and f xx < 0. Thus f(x, y) has a local maximum value at (1, 5).<br />

4. To find the critical points, we solve f x = 0 and f y = 0 for x and y. Solving<br />

f x = 3x 2 − 6x = 0<br />

f y = 2y + 10 = 0<br />

shows that x = 0 or x = 2 and y = −5. There are two critical points: (0, −5) and (2, −5).<br />

We have<br />

D = (f xx)(f yy) − (f xy) 2 = (6x − 6)(2) − (0) 2 = 12x − 12.<br />

When x = 0, we have D = −12 < 0, so f has a saddle point at (0, −5). When x = 2, we have D = 12 > 0 and<br />

f xx = 6 > 0, so f has a local minimum at (2, −5).<br />

5. To find the critical points, we solve f x = 0 and f y = 0 for x and y. Solving<br />

f x = 3x 2 − 3 = 0<br />

f y = 3y 2 − 3 = 0<br />

shows that x = ±1 and y = ±1. There are four critical points: (1, 1), (−1, 1), (1, −1) and (−1, −1).<br />

We have<br />

D = (f xx)(f yy) − (f xy) 2 = (6x)(6y) − (0) 2 = 36xy.<br />

At the points (1, −1) and (−1, 1), we have D = −36 < 0, so f has a saddle point at (1, −1) and (−1, 1). At (1, 1),<br />

we have D = 36 > 0 and f xx = 6 > 0, so f has a local minimum at (1, 1). At (−1, −1), we have D = 36 > 0 and<br />

f xx = −6 < 0, so f has a local maximum at (−1, −1).


1040 Chapter Fifteen /SOLUTIONS<br />

6. To find the critical points, we solve f x = 0 and f y = 0 for x and y. Solving<br />

f x = 3x 2 − 6x = 0<br />

f y = 3y 2 − 3 = 0<br />

shows that x = 0 or x = 2 and y = −1 or y = 1. There are four critical points: (0, −1), (0, 1), (2, −1), and (2, 1).<br />

We have<br />

D = (f xx)(f yy) − (f xy) 2 = (6x − 6)(6y) − (0) 2 = (6x − 6)(6y).<br />

At the point (0, −1), we have D > 0 and f xx < 0, so f has a local maximum.<br />

At the point (0, 1), we have D < 0, so f has a saddle point.<br />

At the point (2, −1), we have D < 0, so f has a saddle point.<br />

At the point (2, 1), we have D > 0 and f xx > 0, so f has a local minimum.<br />

7. To find the critical points, we solve f x = 0 and f y = 0 for x and y. Solving<br />

f x = 3x 2 − 3 = 0<br />

f y = 3y 2 − 12y = 0<br />

shows that x = −1 or x = 1 and y = 0 or y = 4. There are four critical points: (−1, 0), (1, 0), (−1, 4), and (1, 4).<br />

We have<br />

D = (f xx)(f yy) − (f xy) 2 = (6x)(6y − 12) − (0) 2 = (6x)(6y − 12).<br />

At critical point (−1, 0), we have D > 0 and f xx < 0, so f has a local maximum at (−1, 0).<br />

At critical point (1, 0), we have D < 0, so f has a saddle point at (1, 0).<br />

At critical point (−1, 4), we have D < 0, so f has a saddle point at (−1, 4).<br />

At critical point (1, 4), we have D > 0 and f xx > 0, so f has a local minimum at (1, 4).<br />

8. Find the critical point(s) by setting<br />

f x = (xy + 1) + (x + y) · y = y 2 + 2xy + 1 = 0,<br />

f y = (xy + 1) + (x + y) · x = x 2 + 2xy + 1 = 0,<br />

then we get x 2 = y 2 , and so x = y or x = −y.<br />

If x = y, then x 2 + 2x 2 + 1 = 0, that is, 3x 2 = −1, and there is no real solution. If x = −y, then x 2 − 2x 2 + 1 = 0,<br />

which gives x 2 = 1. Solving it we get x = 1 or x = −1, then y = −1 or y = 1, respectively. Hence, (1, −1) and (−1, 1)<br />

are critical points.<br />

Since<br />

f xx(x, y) = 2y,<br />

f xy(x, y) = 2y + 2x<br />

f yy(x, y) = 2x,<br />

and<br />

the discriminant is<br />

D(x, y) = f xxf yy − f 2 xy<br />

= 2y · 2x − (2y + 2x) 2<br />

= −4(x 2 + xy + y 2 ).<br />

thus<br />

Therefore (1, −1) and (−1, 1) are saddle points.<br />

9. At a critical point, f x = 0, f y = 0.<br />

D(1, −1) = −4(1 2 + 1 · (−1) + (−1) 2 ) = −4 < 0,<br />

D(−1, 1) = −4((−1) 2 + (−1) · 1 + 1 2 ) = −4 < 0.<br />

f x = 8y − (x + y) 3 = 0, we know 8y = (x + y) 3 .<br />

f y = 8x − (x + y) 3 = 0, we know 8x = (x + y) 3 .<br />

Therefore we must have x = y. Since (x + y) 3 = (2y) 3 = 8y 3 , this tells us that 8y − 8y 3 = 0. Solving gives y = 0, ±1.<br />

Thus the critical points are (0, 0), (1, 1), (−1, −1).


15.1 SOLUTIONS 1041<br />

f yy = f xx = −3(x + y) 2 , and f xy = 8 − 3(x + y) 2 .<br />

The discriminant is<br />

D(x, y) = f xxf yy − f 2 xy<br />

= 9(x + y) 4 − ( 64 − 48(x + y) 2 + 9(x + y) 4)<br />

= −64 + 48(x + y) 2 .<br />

D(0, 0) = −64 < 0, so (0, 0) is a saddle point.<br />

D(1, 1) = −64 + 192 > 0 and f xx(1, 1) = −12 < 0, so (1, 1) is a local maximum.<br />

D(−1, −1) = −64 + 192 > 0 and f xx(−1, −1) = −12 < 0, so (−1, −1) is a local maximum.<br />

10. To find the critical points, we solve f x = 0 and f y = 0 for x and y. Solving<br />

f x = 6 − 2x + y = 0,<br />

f y = x − 2y = 0.<br />

We see from the second equation that x = 2y. Substituting this into the first equation shows that y = 2. The only critical<br />

point is (4, 2).<br />

We have<br />

D = (f xx)(f yy) − (f xy) 2 = (−2)(−2) − 1 2 = 3.<br />

Since D > 0 and f xx = −2 < 0, the function f has a local maximum at (4, 2).<br />

11. To find the critical points, we solve f x = 0 and f y = 0 for x and y. Solving<br />

shows that the only critical point is (0, 0).<br />

We have<br />

f x = e 2x2 +y 2 (4x) = 0,<br />

f y = e 2x2 +y 2 (2y) = 0,<br />

D = (f xx)(f yy) − (f xy) 2 = e 2x2 +y 2 (4 + (4x) 2 ) · e 2x2 +y 2 (2 + (2y) 2 ) − (e 2x2 +y 2 (4x · 2y)) 2 .<br />

At (0, 0), we have D = 4 · 2 − 0 2 > 0 and f xx = 4 > 0, so the function has a local minimum at the point (0, 0).<br />

12. At the origin f(0, 0) = 0. Since x 6 ≥ 0 and y 6 ≥ 0, the point (0, 0) is a local (and global) minimum. The second<br />

derivative test does not tell you anything since D = 0.<br />

13. At the origin g(0, 0) = 0. Since y 3 ≥ 0 for y > 0 and y 3 < 0 for y < 0, the function g takes on both positive and negative<br />

values near the origin, which must therefore be a saddle point. The second derivative test does not tell you anything since<br />

D = 0.<br />

14. At the origin h(0, 0) = 1. Since cos x and cos y are never above 1, the origin must be a local (and global) maximum. The<br />

second derivative test<br />

D = h xxh yy − (h xy) 2 = ( (− cos x cos y)(− cos x cos y) − (sin x sin y) 2) ∣ ∣<br />

x=0,y=0<br />

= ( cos 2 x cos 2 y − sin 2 x sin 2 y ) ∣ ∣<br />

x=0,y=0<br />

= 1 > 0<br />

and h xx(0, 0) < 0, so (0, 0) is a local maximum.<br />

15. At the origin, the second derivative test gives<br />

Thus k(0, 0) is a saddle point.<br />

D = k xxk yy − (k xy) 2 = ( (− sin x sin y)(− sin x sin y) − (cos x cos y) 2)∣ ∣<br />

x=0,y=0<br />

= sin 2 0 sin 2 0 − cos 2 0 cos 2 0<br />

= −1 < 0.<br />

Problems<br />

16. (a) P is a local maximum.<br />

(b) Q is a saddle point.<br />

(c) R is a local minimum.<br />

(d) S is none of these.


1042 Chapter Fifteen /SOLUTIONS<br />

17. Figure 15.1 shows the gradient vectors around P and Q pointing perpendicular to the contours and in the direction of<br />

increasing values of the function.<br />

y<br />

−2<br />

5<br />

3<br />

1<br />

S<br />

1<br />

6<br />

2<br />

4<br />

−1<br />

−3<br />

−6<br />

−5<br />

−4<br />

−1<br />

✒✿<br />

R<br />

✙ ■ 3<br />

☛ ❘<br />

❲<br />

0 0<br />

✒✒ ■ Q<br />

❘❘ ✠ ❨ ✐✛<br />

✮ ✙<br />

0 0<br />

−6<br />

−5<br />

−4<br />

−6<br />

x<br />

−3<br />

−1<br />

2<br />

4<br />

❘ P<br />

✶<br />

✲ ✠<br />

✐<br />

✛<br />

6<br />

5<br />

3<br />

1<br />

−2<br />

6<br />

Figure 15.1<br />

18. Figure 15.2 shows the direction of ∇f at the points where ‖∇f‖ is largest, since at those points the contours are closest<br />

together.<br />

y<br />

−6<br />

−5<br />

−4<br />

−2<br />

3<br />

1<br />

S<br />

5<br />

✠ R<br />

−1<br />

0 0<br />

0 0<br />

4<br />

2<br />

1<br />

Q<br />

−1<br />

−3<br />

−5<br />

−6<br />

−6<br />

−4<br />

x<br />

−3<br />

−1<br />

P<br />

−2<br />

2<br />

4<br />

6<br />

5<br />

6<br />

3<br />

1<br />

Figure 15.2<br />

19. First, we identify the critical points. The partials are f x(x, y) = 3x 2 and f y(x, y) = −2ye −y2 . These will vanish<br />

simultaneously when x = 0 and y = 0, so our only critical point is (0, 0). The discriminant is<br />

D = f xx(x, y)f yy(x, y) − f 2 xy(x, y) = (6x)(4y 2 e −y2 − 2e −y2 ) − 0 = 12xe −y2 (2y 2 − 1).<br />

Unfortunately, the discriminant is zero at the origin so the second derivative test can tell us nothing about our critical point.<br />

We can, however, see that we are at a saddle point by looking at the behavior of f(x, y) along the line y = 0. Here we have<br />

f(x, 0) = x 3 + 1, so for positive x, we have f(x, 0) > 1 = f(0, 0) and for negative x, we have f(x, 0) < 1 = f(0, 0).<br />

So f(x, y) has neither a maximum nor a minimum at (0, 0).


15.1 SOLUTIONS 1043<br />

20. To find critical points, set partial derivatives equal to zero:<br />

The critical points are<br />

To classify, calculate D = E xxE yy − (E xy) 2 = cos x.<br />

At the points (0, 0), (±2π, 0), (±4π, 0), (±6π, 0), · · ·<br />

E x = sin x = 0 when x = 0, ±π, ±2π, · · ·<br />

E y = y = 0 when y = 0.<br />

· · · (−2π, 0), (−π, 0), (0, 0), (π, 0), (2π, 0), (3π, 0) · · ·<br />

D = (1) > 0 and E xx > 0 (SinceE xx(0, 2kπ) = cos(2kπ) = 1).<br />

Therefore (0, 0), (±2π, 0), (±4π, 0), (±6π, 0), · · · are local minima.<br />

At the points (±π, 0), (±3π, 0), (±5π, 0), (±7π, 0), · · ·, we have cos(2k + 1)π = −1, so<br />

D = (−1) < 0.<br />

Therefore (±π, 0), (±3π, 0), (±5π, 0), (±7π, 0), · · · are saddle points.<br />

21. To find the critical points, we must solve the equations<br />

The first equation has solution<br />

The second equation has solution<br />

Since x can be anything, the lines<br />

are lines of critical points.<br />

We calculate<br />

∂f<br />

∂x = ex (1 − cos y) = 0<br />

∂f<br />

∂y = ex (sin y) = 0.<br />

y = 0, ±2π, ±4π, . . . .<br />

y = 0, ±π, ±2π, ±3π, . . . .<br />

y = 0, ±2π, ±4π, . . .<br />

D = (f xx)(f yy) − (f xy) 2 = e x (1 − cos y)e x cos y − (e x sin y) 2<br />

= e 2x (cos y − cos 2 y − sin 2 y)<br />

= e 2x (cos y − 1)<br />

At any critical point on one of the lines y = 0, y = ±2π, y = ±4π, . . .,<br />

D = e 2x (1 − 1) = 0.<br />

Thus, D tells us nothing. However, all along these critical lines, the value of the function, f, is zero. Since the function f<br />

is never negative, the critical points are all both local and global minima.<br />

22. At a critical point,<br />

and<br />

f x = cos x sin y = 0 so cos x = 0 or sin y = 0;<br />

Case 1: Assume cos x = 0. This gives<br />

f y = sin x cos y = 0 so sin x = 0 or cos y = 0.<br />

x = · · · − 3π 2 , − π 2 , π 2 , 3π 2 , · · ·<br />

(This can be written more compactly as: x = kπ + π/2, for k = 0, ±1, ±2, · · ·.)<br />

If cos x = 0, then sin x = ±1 ≠ 0. Thus in order to have f y = 0 we need cos y = 0, giving<br />

y = · · · − 3π 2 , − π 2 , π 2 , 3π 2 , · · ·


1044 Chapter Fifteen /SOLUTIONS<br />

(More compactly, y = lπ + π/2, for l = 0, ±1, ±2, · · ·)<br />

Case 2: Assume sin y = 0. This gives<br />

y = · · · − 2π, −π, 0, π, 2π, · · ·<br />

(More compactly, y = lπ, for l = 0, ±1, ±2, · · ·)<br />

If sin y = 0, then cos y = ±1 ≠ 0, so to get f y = 0 we need sin x = 0, giving<br />

x = · · · , −2π, −π, 0, π, 2π, · · ·<br />

(More compactly, x = kπ for k = 0, ±0, ±1, ±2, · · ·)<br />

Hence we get all the critical points of f(x, y). Those from Case 1 are as follows:<br />

(<br />

· · · − π 2 , − π ) (<br />

, − π 2 2 , π ) (<br />

, − π 2 2 , 3π 2<br />

Those from Case 2 are as follows:<br />

· · ·<br />

More compactly these points can be written as,<br />

( π<br />

· · ·<br />

2 , − π )<br />

2<br />

( 3π<br />

2 , − π 2<br />

)<br />

,<br />

( π<br />

,<br />

2 , π )<br />

2<br />

( 3π<br />

2 , π 2<br />

)<br />

,<br />

( π<br />

,<br />

2 , 3π )<br />

2<br />

( 3π<br />

2 , 3π 2<br />

)<br />

· · ·<br />

· · ·<br />

)<br />

· · ·<br />

· · · (−π, −π), (−π, 0), (−π, π), (−π, 2π) · · ·<br />

· · · (0, −π), (0, 0), (0, π), (0, 2π) · · ·<br />

· · · (π, −π), (π, 0), (π, π), (π, 2π) · · ·<br />

(kπ, lπ), for k = 0, ±1, ±2, · · · , l = 0, ±1, ±2, · · ·<br />

and (kπ + π 2 , lπ + π ), for k = 0, ±1, ±2, · · · , l = 0, ±1, ±2, · · ·<br />

2<br />

To classify the critical points, we find the discriminant. We have<br />

f xx = − sin x sin y, f yy = − sin x sin y, and f xy = cos x cos y.<br />

Thus the discriminant is<br />

D(x, y) = f xxf yy − f 2 xy<br />

= (− sin x sin y)(− sin x sin y) − (cos x cos y) 2<br />

= sin 2 x sin 2 y − cos 2 x cos 2 y<br />

= sin 2 y − cos 2 x. (Use: sin 2 x = 1 − cos 2 x and factor.)<br />

At points of the form (kπ, lπ) where k = 0, ±1, ±2, · · · ; l = 0, ±1, ±2, · · ·, we have<br />

D(x, y) = −1 < 0 so (kπ, lπ) are saddle points.<br />

At points of the form (kπ + π , lπ + π ) where k = 0, ±1, ±2, · · · ; l = 0, ±1, ±2, · · ·<br />

2 2<br />

D(kπ + π , lπ + π ) = 1 > 0, so we have two cases:<br />

2 2<br />

If k and l are both even or k and l are both odd, then<br />

f xx = − sin x sin y = −1 < 0, so (kπ + π , lπ + π ) are local maximum points.<br />

2 2<br />

If k is even but l is odd or k is odd but l is even, then<br />

f xx = 1 > 0 so (kπ + π , lπ + π ) are local minimum points.<br />

2 2<br />

23. At a local maximum value of f,<br />

∂ f<br />

= −2x − B = 0.<br />

∂ x<br />

We are told that this is satisfied by x = −2. So −2(−2) − B = 0 and B = 4. In addition,<br />

∂ f<br />

∂ y = −2y − C = 0<br />

and we know this holds for y = 1, so −2(1) − C = 0, giving C = −2. We are also told that the value of f is 15 at the<br />

point (−2, 1), so<br />

15 = f(−2, 1) = A − ((−2) 2 + 4(−2) + 1 2 − 2(1)) = A − (−5), so A = 10.


and<br />

15.1 SOLUTIONS 1045<br />

Now we check that these values of A, B, and C give f(x, y) a local maximum at the point (−2, 1). Since<br />

f xx(−2, 1) = −2,<br />

f yy(−2, 1) = −2<br />

f xy(−2, 1) = 0,<br />

we have that f xx(−2, 1)f yy(−2, 1) − f 2 xy(−2, 1) = (−2)(−2) − 0 > 0 and f xx(−2, 1) < 0. Thus, f has a local<br />

maximum value 15 at (−2, 1).<br />

24. (a) (1, 3) is a critical point. Since f xx > 0 and the discriminant<br />

the point (1, 3) is a minimum.<br />

(b) See Figure 15.3.<br />

D = f xxf yy − f 2 xy = f xxf yy − 0 2 = f xxf yy > 0,<br />

y<br />

0<br />

y<br />

120<br />

64<br />

32<br />

16<br />

4<br />

1<br />

3<br />

1<br />

✠<br />

x<br />

−7<br />

−5<br />

1<br />

−3<br />

0<br />

1<br />

1<br />

3 579<br />

−1<br />

0<br />

1<br />

0<br />

0<br />

5 79<br />

3<br />

−1<br />

−3<br />

−5<br />

−7<br />

x<br />

Figure 15.3<br />

Figure 15.4<br />

25. (a) (a, b) is a critical point. Since the discriminant D = f xxf yy − f 2 xy = −f 2 xy < 0, (a, b) is a saddle point.<br />

(b) See Figure 15.4.<br />

26. Begin by constructing little pieces of the contour diagram around each of the points (−1, 0), (3, 3), and (3, −3) where<br />

some information about f is given. The general shape will be as in Figure 15.5, and the directions of increasing contour<br />

values are indicated for each part. Then complete the diagram in any way. One possible solution is given in Figure 15.6.<br />

0 1<br />

2<br />

0 1 2<br />

3<br />

4<br />

Figure 15.5: Part of contour<br />

diagram with arrows showing<br />

direction of increasing<br />

function values<br />

Figure 15.6: Contour diagram<br />

of f(x, y)


1046 Chapter Fifteen /SOLUTIONS<br />

27. Since (2, 4) is a local minimum, the contours around (2, 4) are closed curves with increasing values as we go away from<br />

the point (2, 4). We assume that the function values continue to increase as we move parallel to the y-axis to the point<br />

(2, 1). Since (2, 1) is a saddle point, we draw the contours so that the values go down as we move up or down from this<br />

point, and up as we move to the left or right. One possible contour diagram is shown in Figure 15.7.<br />

y<br />

y<br />

6<br />

5<br />

f = −1<br />

4<br />

3<br />

2<br />

1<br />

−4<br />

−3<br />

−2<br />

−1<br />

0 1 2 3 4<br />

f = 1<br />

(0, 0)<br />

f = 1<br />

f = 0<br />

f = 0<br />

x<br />

−1<br />

−1 1 2 3 4 5 6<br />

−1<br />

x<br />

f = −1<br />

Figure 15.7<br />

Figure 15.8<br />

28. (a) Setting the partial derivatives equal to 0, we have<br />

f x(x, y) = 2x(x 2 + y) + 2x(x 2 − y) = 4x 3 = 0<br />

f y(x, y) = −(x 2 + y) + (x 2 − y) = −2y = 0.<br />

Thus, (0, 0) is the only critical point.<br />

(b) Calculating D gives<br />

D = (f xx)(f yy) − (f xy) 2 = (12x 2 )(−2) − 0 2 = −24x 2 .<br />

At x = 0, y = 0, we have<br />

D(0, 0) = 0.<br />

Thus the second derivative test tells us nothing about the nature of the critical point.<br />

(c) Since f(0, 0) = 0, we sketch contours with values near 0. The contour f = 0 is given by<br />

that is, the two parabolas<br />

(x 2 − y)(x 2 + y) = 0,<br />

y = x 2 and y = −x 2<br />

We also sketch the contours f = 1 and f = −1. See Figure 15.8.<br />

Since there are values of the function which are both positive (above f(0, 0)) and negative (below (f(0, 0)),<br />

near the critical point (0, 0), the origin is neither a local maximum nor a local minimum; it is a saddle point.<br />

29. We have f x = 2kx − 4y and f y = 2y − 4x, so f xx = 2k, f xy = −4, and f yy = 2. The discriminant is<br />

D = (f xx)(f yy) − (f xy) 2 = (2k)(2) − (−4) 2 = 4k − 16.<br />

Since D = 4k − 16, we see that D < 0 when k < 4. The function has a saddle point at the point (0, 0) when k < 4.<br />

When k > 4, we have D > 0 and f xx > 0, so the function has a local minimum at the point (0, 0). When k = 4,<br />

the discriminant is zero, and we get no information about this critical point. By looking at the values of the function in<br />

Table 15.1, it appears that f has a local minimum at the point (0, 0) when k = 4.<br />

Table 15.1<br />

y<br />

−0.1 0 0.1<br />

−0.1 0.01 0.01 0.09<br />

0 0.04 0 0.04<br />

0.1 0.09 0.01 0.01<br />

x<br />

(a) The function f(x, y) has a saddle point at (0, 0) if k < 4.


15.1 SOLUTIONS 1047<br />

(b) There are no values of k for which this function has a local maximum at the point (0, 0).<br />

(c) The function f(x, y) has a local minimum at (0, 0) if k ≥ 4.<br />

30. The first order partial derivatives are<br />

And the second order partial derivatives are<br />

f x(x, y) = 2kx − 2y and f y(x, y) = 2ky − 2x.<br />

f xx(x, y) = 2k f xy(x, y) = −2 f yy(x, y) = 2k<br />

Since f x(0, 0) = f y(0, 0) = 0, the point (0, 0) is a critical point. The discriminant is<br />

D = (2k)(2k) − 4 = 4(k 2 − 1).<br />

For k = ±2, the discriminant is positive, D = 12. When k = 2, f xx(0, 0) = 4 which is positive so we have a local<br />

minimum at the origin. When k = −2, f xx(0, 0) = −4 so we have a local maximum at the origin. In the case k = 0,<br />

D = −4 so the origin is a saddle point.<br />

Lastly, when k = ±1 the discriminant is zero, so the second derivative test can tell us nothing. Luckily, we can factor<br />

f(x, y) when k = ±1. When k = 1,<br />

f(x, y) = x 2 − 2xy + y 2 = (x − y) 2 .<br />

This is always greater than or equal to zero. So f(0, 0) = 0 is a minimum and the surface is a trough-shaped parabolic<br />

cylinder with its base along the line x = y.<br />

When k = −1,<br />

f(x, y) = −x 2 − 2xy − y 2 = −(x + y) 2 .<br />

This is always less than or equal to zero. So f(0, 0) = 0 is a maximum. The surface is a parabolic cylinder, with its top<br />

ridge along the line x = −y.<br />

k = −2<br />

y<br />

k = −1<br />

y k = 0<br />

y<br />

−16<br />

−12<br />

−8<br />

−4<br />

−1<br />

x<br />

−1<br />

−5<br />

−10<br />

−20<br />

−30<br />

−30<br />

−20<br />

−10<br />

−5<br />

−1<br />

x<br />

16<br />

12<br />

8<br />

4<br />

1<br />

−1<br />

−4<br />

−8<br />

−12<br />

−16<br />

−1<br />

1<br />

−4<br />

4<br />

−16<br />

−12<br />

−8<br />

8<br />

12<br />

16<br />

x<br />

k = 1<br />

y<br />

k = 2<br />

y<br />

30<br />

20<br />

10<br />

5<br />

1<br />

1<br />

x<br />

1<br />

4<br />

8<br />

16<br />

12<br />

x<br />

5<br />

10<br />

20<br />

30<br />

Figure 15.9


1048 Chapter Fifteen /SOLUTIONS<br />

31. The partial derivatives are<br />

f x(x, y) = 3x 2 − 3y 2 and f y(x, y) = −6xy.<br />

Now f x(x, y) will vanish if x = ±y and f y(x, y) will vanish if either x = 0 or y = 0. Since the partial derivatives<br />

are defined everywhere, the only critical points are where f x(x, y) and f y(x, y) vanish simultaneously. (0, 0) is the only<br />

critical point.<br />

To find the contour for f(x, y) = 0, we solve the equation x 3 − 3xy 2 = 0. This can be factored into<br />

f(x, y) = x(x − √ 3 y)(x + √ 3 y) = 0<br />

whose roots are x = 0, x = √ 3 y and x = − √ 3 y. Each of these roots describes a line through the origin; the three of<br />

them divide the plane into six regions. Crossing any one of these lines will change the sign of only one of the three factors<br />

of f(x, y), which will change the sign of f(x, y).<br />

y<br />

f > 0<br />

f < 0<br />

2<br />

11<br />

−1<br />

−2<br />

✛ y = x<br />

√<br />

3<br />

−2<br />

f < 0<br />

0<br />

−1<br />

0 0<br />

1 2<br />

0 0<br />

f > 0<br />

x<br />

2<br />

11<br />

−1<br />

−2<br />

✛ y = −x<br />

√<br />

3<br />

f > 0 f < 0<br />

Figure 15.10<br />

Solutions for Section 15.2<br />

Exercises<br />

1. Mississippi lies entirely within a region designated as 80s so we expect both the maximum and minimum daily high<br />

temperatures within the state to be in the 80s. The southwestern-most corner of the state is close to a region designated as<br />

90s, so we would expect the temperature here to be in the high 80s, say 87-88. The northern-most portion of the state is<br />

located near the center of the 80s region. We might expect the high temperature there to be between 83-87.<br />

Alabama also lies completely within a region designated as 80s so both the high and low daily high temperatures<br />

within the state are in the 80s. The southeastern tip of the state is close to a 90s region so we would expect the temperature<br />

here to be about 88-89 degrees. The northern-most part of the state is near the center of the 80s region so the temperature<br />

there is 83-87 degrees.<br />

Pennsylvania is also in the 80s region, but it is touched by the boundary line between the 80s and a 70s region. Thus<br />

we expect the low daily high temperature to occur there and be about 80 degrees. The state is also touched by a boundary<br />

line of a 90s region so the high will occur there and be 89-90 degrees.<br />

New York is split by a boundary between an 80s and a 70s region, so the northern portion of the state is likely to be<br />

about 74-76 while the southern portion is likely to be in the low 80s, maybe 81-84 or so.<br />

California contains many different zones. The northern coastal areas will probably have the daily high as low as<br />

65-68, although without another contour on that side, it is difficult to judge how quickly the temperature is dropping off<br />

to the west. The tip of Southern California is in a 100s region, so there we expect the daily high to be 100-101.<br />

Arizona will have a low daily high around 85-87 in the northwest corner and a high in the 100s, perhaps 102-107 in<br />

its southern regions.<br />

Massachusetts will probably have a high daily high around 81-84 and a low daily high of 70.


15.2 SOLUTIONS 1049<br />

2. Let the line be in the form y = b + mx. When x equals −1, 0 and 1, then y equals b − m, b, and b + m, respectively. The<br />

sum of the squares of the vertical distances, which is what we want to minimize, is<br />

f(m, b) = (2 − (b − m)) 2 + (−1 − b) 2 + (1 − (b + m)) 2 .<br />

To find the critical points, we compute the partial derivatives with respect to m and b,<br />

f m = 2(2 − b + m) + 0 + 2(1 − b − m)(−1)<br />

= 4 − 2b + 2m − 2 + 2b + 2m<br />

= 2 + 4m,<br />

f b = 2(2 − b + m)(−1) + 2(−1 − b)(−1) + 2(1 − b − m)(−1)<br />

= −4 + 2b − 2m + 2 + 2b − 2 + 2b + 2m<br />

= −4 + 6b.<br />

Setting both partial derivatives equal to zero, we get a system of equations:<br />

2 + 4m = 0,<br />

−4 + 6b = 0.<br />

The solution is m = −1/2 and b = 2/3. One can check that it is a minimum. Hence, the regression line is y = 2 3 − 1 2 x.<br />

3. The function f has no global maximum or global minimum.<br />

4. The function g has a global minimum (it is 0) but no global maximum.<br />

5. The function h has no global maximum or minimum.<br />

6. Since f(x, y) ≤ 0 for all x, y and sincef(0, 0) = 0, the function has a global maximum (it is 0) and no global minimum.<br />

7. Suppose x is fixed. Then for large values of y the sign of f is determined by the highest power of y, namely y 3 . Thus,<br />

So f does not have a global maximum or minimum.<br />

f(x, y) → ∞ as y → ∞<br />

f(x, y) → −∞ as y → −∞.<br />

8. To maximize z = x 2 + y 2 , it suffices to maximize x 2 and y 2 . We can maximize both of these at the same time by<br />

taking the point (1, 1), where z = 2. It occurs on the boundary of the square. (Note: We also have maxima at the points<br />

(−1, −1), (−1, 1) and (1, −1) which are on the boundary of the square.)<br />

To minimize z = x 2 + y 2 , we choose the point (0, 0), where z = 0. It does not occur on the boundary of the square.<br />

9. To maximize z = −x 2 − y 2 it suffices to minimize x 2 and y 2 . Thus, the maximum is at (0, 0), where z = 0. It does not<br />

occur on the boundary of the square.<br />

To minimize z = −x 2 − y 2 , it suffices to maximize x 2 and y 2 . Do this by taking the point (1, 1), (−1, −1), (−1, 1),<br />

or (1, −1) where z = −2. These occur on the boundary of the square.<br />

10. To maximize this function, it suffices to maximize x 2 and minimize y 2 . We can do this by choosing the point (1, 0), or<br />

(−1, 0) where z = 1. These occur on the boundary of the square.<br />

To minimize z = x 2 − y 2 , it suffices to maximize y 2 and minimize x 2 . We can do this by taking the point (0, 1), or<br />

(0, −1) where z = −1. These occur on the boundary of the square.<br />

11. The maximum value, which is slightly above 30, say 30.5, occurs approximately at the origin. The minimum value, which<br />

is about 20.5, occurs at (2.5, 5).<br />

12. The maxima occur at about (π/2, 0) and (π/2, 2π). The minimum occurs at (π/2, π). The maximum value is about 1,<br />

the minimum value is about −1.<br />

13. The maximum value, which is about 11, occurs at (5.1, 4.9). The minimum value, which is about −1, occurs at (1, 3.9).<br />

Problems<br />

14. To find critical points of g we solve<br />

g x(x, y) = Ax + By − D = 0<br />

g y(x, y) = Bx + Cy − E = 0


1050 Chapter Fifteen /SOLUTIONS<br />

which is equivalent to<br />

Ax + By = D<br />

Bx + Cy = E.<br />

The second derivative test shows that the critical point gives a minimum for g because g xxg yy − g 2 xy = AC − B 2 > 0<br />

and g xx = A > 0.<br />

15. The variables are a and b, so we set<br />

so, collecting terms and dividing by 4 and 2 respectively,<br />

∂S<br />

= 2(a + b) + 8(4a + b − 2) + 18(9a + b − 4) = 0<br />

∂a<br />

∂S<br />

= 2(a + b) + 2(4a + b − 2) + 2(9a + b − 4) = 0,<br />

∂b<br />

49a + 7b − 22 = 0<br />

14a + 3b − 6 = 0.<br />

Solving gives a = 24/49, b = −2/7.<br />

Since there is only one critical point and S is unbounded as a, b → ∞, this critical point is the global minimum.<br />

Therefore, the best fitting parabola is<br />

y = 24<br />

49 x2 − 2 7 .<br />

16. We must find b and m to minimize the function<br />

g(b, m) =<br />

=<br />

∫ 1<br />

0<br />

∫ 1<br />

0<br />

(x 2 − (b + mx)) 2 dx<br />

(x 4 − 2bx 2 + b 2 − 2mx 3 + 2bmx + m 2 x 2 )dx<br />

= 1 5 − 2 3 b + b2 − m 2<br />

The critical points of g are given by the equations<br />

+ bm +<br />

m2<br />

3 .<br />

∂g<br />

∂b = − 2 3 + 2b + m = 0<br />

∂g<br />

∂m = − 1 2 + b + 2 3 m = 0.<br />

The solution is b = −1/6 and m = 1. The linear least squares approximation of x 2 on the interval [0, 1] is L(x, y) =<br />

−1/6 + x.<br />

17. We must find b and m to minimize the function<br />

g(b, m) =<br />

=<br />

∫ 1<br />

0<br />

∫ 1<br />

0<br />

(x 3 − (b + mx)) 2 dx<br />

(x 6 − 2bx 3 + b 2 − 2mx 4 + 2bmx + m 2 x 2 )dx<br />

= 1 6 − 1 2 b + b2 − 2m 5<br />

The critical points of g are given by the equations<br />

+ bm +<br />

m2<br />

3 .<br />

∂g<br />

∂b = − 1 2 + 2b + m = 0<br />

∂g<br />

∂m = − 2 5 + b + 2 3 m = 0.<br />

The solution is b = −1/5 and m = 9/10. The linear least squares approximation of x 3 on the interval [0, 1] is L(x, y) =<br />

−0.2 + 0.9x.


15.2 SOLUTIONS 1051<br />

18. We calculate the partial derivatives and set them to zero.<br />

∂ (range)<br />

∂t<br />

∂ (range)<br />

∂h<br />

= −10t − 6h + 400 = 0<br />

= −6t − 6h + 300 = 0.<br />

solving we obtain<br />

so<br />

10t + 6h = 400<br />

6t + 6h = 300<br />

4t = 100<br />

t = 25<br />

Solving for h, we obtain 6h = 150, yielding h = 25. Since the range is quadratic in h and t, the second derivative test<br />

tells us this is a local and global maximum. So the optimal conditions are h = 25% humidity and t = 25 ◦ C.<br />

19. (a) This tells us that an increase in the price of either product causes a decrease in the quantity demanded of both<br />

products. An example of products with this relationship is tennis rackets and tennis balls. An increase in the price of<br />

either product is likely to lead to a decrease in the quantity demanded of both products as they are used together. In<br />

economics, it is rare for the quantity demanded of a product to increase if its price increases, so for q 1, the coefficient<br />

of p 1 is negative as expected. The coefficient of p 2 in the expression could be either negative or positive. In this case,<br />

it is negative showing that the two products are complementary in use. If it were positive, however, it would indicate<br />

that the two products are competitive in use, for example Coke and Pepsi.<br />

(b) The revenue from the first product would be q 1p 1 = 150p 1 − 2p 2 1 − p 1p 2, and the revenue from the second product<br />

would be q 2p 2 = 200p 2 − p 1p 2 − 3p 2 2. The total sales revenue of both products, R, would be<br />

R(p 1, p 2) = 150p 1 + 200p 2 − 2p 1p 2 − 2p 2 1 − 3p 2 2.<br />

Note that R is a function of p 1 and p 2. To find the critical points of R, set ∇R = 0, i.e.,<br />

This gives<br />

and<br />

∂R<br />

∂p 1<br />

= ∂R<br />

∂p 2<br />

= 0.<br />

∂R<br />

∂p 1<br />

= 150 − 2p 2 − 4p 1 = 0<br />

∂R<br />

∂p 2<br />

= 200 − 2p 1 − 6p 2 = 0<br />

Solving simultaneously, we have p 1 = 25 and p 2 = 25. Therefore the point (25, 25) is a critical point for R. Further,<br />

∂ 2 R<br />

∂p 2 1<br />

so the discriminant at this critical point is<br />

= −4, ∂2 R<br />

∂p 2 2<br />

= −6,<br />

∂ 2 R<br />

∂p 1∂p 2<br />

= −2,<br />

D = (−4)(−6) − (−2) 2 = 20.<br />

Since D > 0 and ∂ 2 R/∂p 2 1 < 0, this critical point is a local maximum. Since R is quadratic in p 1 and p 2, this is a<br />

global maximum. Therefore the maximum possible revenue is<br />

R = 150(25) + 200(25) − 2(25)(25) − 2(25) 2 − 3(25) 2<br />

= (6)(25) 2 + 8(25) 2 − 7(25) 2<br />

= 4375.<br />

This is obtained when p 1 = p 2 = 25. Note that at these prices, q 1 = 75 units, and q 2 = 100 units.


1052 Chapter Fifteen /SOLUTIONS<br />

20. The total revenue is<br />

and as q = q 1 + q 2, this gives<br />

R = pq = (60 − 0.04q)q = 60q − 0.04q 2 ,<br />

R = 60q 1 + 60q 2 − 0.04q 2 1 − 0.08q 1q 2 − 0.04q 2 2.<br />

Therefore, the profit is<br />

P (q 1, q 2) = R − C 1 − C 2<br />

At a local maximum point, we have grad P = ⃗0 :<br />

Solving these equations, we find that<br />

= −13.7 + 60q 1 + 60q 2 − 0.07q 2 1 − 0.08q 2 2 − 0.08q 1q 2.<br />

∂P<br />

∂q 1<br />

= 60 − 0.14q 1 − 0.08q 2 = 0,<br />

∂P<br />

∂q 2<br />

= 60 − 0.16q 2 − 0.08q 1 = 0.<br />

q 1 = 300 and q 2 = 225.<br />

To see whether or not we have found a local maximum, we compute the second-order partial derivatives:<br />

Therefore,<br />

∂ 2 P<br />

∂q 2 1<br />

D = ∂2 P<br />

∂q 2 1<br />

= −0.14,<br />

∂ 2 P<br />

∂q 2 2<br />

−<br />

∂ 2 P<br />

∂q 2 2<br />

= −0.16,<br />

∂ 2 P<br />

∂q 1∂q 2<br />

= −0.08.<br />

∂2 P<br />

∂q 1∂q 2<br />

= (−0.14)(−0.16) − (−0.08) 2 = 0.016,<br />

and so we have found a local maximum point. The graph of P (q 1, q 2) has the shape of an upside down paraboloid since<br />

P is quadratic in q 1 and q 2, hence (300, 225) is a global maximum point.<br />

21. (a) The revenue R = p 1q 1 + p 2q 2. Profit = P = R − C = p 1q 1 + p 2q 2 − 2q 2 1 − 2q 2 2 − 10.<br />

∂P<br />

= p 1 − 4q 1 = 0<br />

∂q 1<br />

∂P<br />

= p 2 − 4q 2 = 0<br />

∂q 2<br />

gives q 1 = p1<br />

4<br />

gives q 2 = p2<br />

4<br />

Since ∂2 P<br />

∂q 2 1<br />

and ∂2 P<br />

∂q 2 1<br />

= −4, ∂2 P<br />

= −4 and ∂2 P<br />

∂q<br />

2<br />

2 ∂q 1 ∂q 2<br />

= 0, at (p 1/4, p 2/4) we have that the discriminant, D = (−4)(−4) > 0<br />

< 0, thus P has a local maximum value at (q 1, q 2) = (p 1/4, p 2/4). Since P is quadratic in q 1 and q 2, this<br />

is a global maximum. So P = p2 1<br />

4<br />

+ p2 2<br />

4<br />

− 2 p2 1<br />

16 − 2 p2 2<br />

16 − 10 = p2 1<br />

8<br />

+ p2 2<br />

8<br />

− 10 is the maximum profit.<br />

(b) The rate of change of the maximum profit as p 1 increases is<br />

∂<br />

∂p 1<br />

(max P ) = 2p1<br />

8 = p1<br />

4 .<br />

22. We want to maximize the theater’s profit, P , as a function of the two variables (prices) p c and p a. As always, P = R − C,<br />

where R is the revenue, R = q cp c + q ap a, and C is the cost, which is of the form C = k(q c + q a) for some constant k.<br />

Thus,<br />

P (p c, p a) = q cp c + q ap a − k(q c + q a)<br />

= rp −3<br />

c<br />

− krp −4<br />

c<br />

+ sp −1<br />

a<br />

− ksp −2<br />

a<br />

To find the critical points, solve<br />

We get p c = 4k/3 and p a = 2k.<br />

∂P<br />

= −3rp −4<br />

c + 4krp −5<br />

c = 0<br />

∂p c<br />

∂P<br />

= −sp −2<br />

a + 2ksp −3<br />

a = 0.<br />

∂p a


15.2 SOLUTIONS 1053<br />

This critical point is a global maximum by the following useful, general argument. Suppose that F (x, y) = f(x) +<br />

g(y), where f has a global maximum at x = b and g has a global maximum at y = d. Then for all x, y:<br />

so F has global maximum at x = b, y = d.<br />

The profit function in this problem has the form<br />

F (x, y) = f(x) + g(y) ≤ f(b) + g(d) = F (b, d),<br />

P (p c, p a) = f(p c) + g(p a),<br />

and the usual single-variable calculus argument using f ′ and g ′ shows that p c = 4k/3 and p a = 2k are global maxima<br />

for f and g, respectively. Thus the maximum profit occurs when p c = 4k/3 and p a = 2k. Thus,<br />

p c<br />

= 4k/3<br />

p a 2k = 2 3 .<br />

23. Let the sides be x, y, z cm. Then the volume is given by V = xyz = 32.<br />

The surface area S is given by<br />

S = 2xy + 2xz + 2yz.<br />

Substituting z = 32/(xy) gives<br />

At a critical point,<br />

S = 2xy + 64<br />

y + 64 x .<br />

∂S<br />

∂x<br />

∂S<br />

∂y<br />

= 2y −<br />

64<br />

x 2 = 0<br />

= 2x −<br />

64<br />

y 2 = 0,<br />

The symmetry of the equations (or by dividing the equations) tells us that x = y and<br />

2x − 64<br />

x 2 = 0<br />

x 3 = 32<br />

x = 32 1/3 = 3.17 cm.<br />

Thus the only critical point is x = y = (32) 1/3 cm and z = 32/ ( (32) 1/3 · (32) 1/3) = (32) 1/3 cm. At the critical point<br />

S xxS yy − (S xy) 2 = 128<br />

x 3 · 128<br />

y 3 − 22 = (128)2<br />

x 3 y 3 − 4.<br />

Since D > 0 and S xx > 0 at this critical point, the critical point x = y = z = (32) 1/3 is a local minimum. Since<br />

S → ∞ as x, y → ∞, the local minimum is a global minimum.<br />

24. Let the sides of the base be x and y cm. Let the height be z cm. Then the volume is given by xyz = 32 and the surface<br />

area, S, is given by<br />

S = xy + 2xz + 2yz.<br />

Substituting z = 32/(xy) gives<br />

At a critical point<br />

The symmetry of the equations tells us that x = y and<br />

S = xy + 64<br />

y + 64 x .<br />

∂S<br />

∂x = y − 64<br />

x = 0 2<br />

∂S<br />

∂y = x − 64<br />

y = 0. 2<br />

x − 64<br />

x 2 = 0<br />

x 3 = 64<br />

x = 4 cm.


1054 Chapter Fifteen /SOLUTIONS<br />

Thus the only critical point is x = y = 4 cm and z = 32/(4 · 4) = 2 cm. At the critical point<br />

D = S xxS yy − (S xy) 2 = 128<br />

x 3 · 128<br />

y 3 − 12 = (128)2<br />

x 3 y 3 − 1.<br />

Since D > 0 and S xx > 0 at this critical point, the critical point x = y = 4, z = 2 is a local minimum. Since S → ∞ as<br />

x, y → ∞, the local minimum is a global minimum.<br />

25. If the coordinates of the corner on the plane are (x, y, z), the volume of the box is V = xyz. Since z = 1 − 3x − 2y on<br />

the plane, the volume is given by<br />

26.<br />

V = xy(1 − 3x − 2y) = xy − 3x 2 y − 2xy 2 .<br />

The domain is the triangular region 0 ≤ x ≤ 1 , 0 ≤ y ≤ (1 − 3x)/2. At a critical point,<br />

3<br />

∂V<br />

∂x = y − 6xy − 2y2 = y(1 − 6x − 2y) = 0<br />

∂V<br />

∂y = x − 3x2 − 4xy = x(1 − 3x − 4y) = 0,<br />

One solution is x = y = 0. Another is x = 0, y = 1 ; another is y = 0, x = 1 . Another is the solution of<br />

2 3<br />

1 − 6x − 2y = 0<br />

1 − 3x − 4y = 0,<br />

namely x = 1 9 , y = 1 6 .<br />

If either x = 0 or y = 0, then V = 0, so these solutions do not give the maximum volume. Since<br />

D = V xxV yy − (V xy) 2 = (−6y)(−4x) − (1 − 6x − 4y) 2<br />

( 1<br />

D<br />

9 , 1 (<br />

= −6 ·<br />

6) 1<br />

6<br />

) (<br />

−4 · 1 ) (<br />

− 1 − 6 · 1<br />

9<br />

9 − 4 · 1 ) 2<br />

= 4 6 9 − 1 9 = 1 3 > 0,<br />

and V xx( 1 9 , 1 6 ) = −1 < 0, the point x = 1 9 , y = 1 6 , is a local maximum at which V = (1/9)(1/6) − 3(1/9)2 (1/6) −<br />

2(1/9)(1/6) 2 = 1/162.<br />

Since all points on the boundary of the domain give V = 0, the local maximum is a global maximum.<br />

h<br />

w<br />

Figure 15.11<br />

l<br />

Let w, h and l be width, height and length of the suitcase in cm. Then its volume V = lwh, and w + h + l ≤ 135.<br />

To maximize the volume V , choose w + h + l = 135, and thus l = 135 − w − h,<br />

Differentiating gives<br />

Find the critical points by solving V w = 0 and V h = 0:<br />

V = wh(135 − w − h)<br />

= 135wh − w 2 h − wh 2<br />

V w = 135h − 2wh − h 2 ,<br />

V h = 135w − w 2 − 2wh.<br />

V w = 0 gives 135h − h 2 = 2wh,<br />

V h = 0 gives 135w − w 2 = 2wh.


15.2 SOLUTIONS 1055<br />

As hw ≠ 0, we cancel h (and w respectively) in the above equations and get<br />

135 − h = 2w<br />

135 − w = 2h<br />

27.<br />

Subtracting gives<br />

w − h = 2(w − h)<br />

hence w = h. Therefore, substituting into the equation V w = 0<br />

and therefore<br />

Since h ≠ 0, we have<br />

135h − h 2 = 2h 2<br />

3h 2 = 135h.<br />

h = 135<br />

3 = 45.<br />

So w = h = 45 cm. Thus, l = 135 − w − h = 45 cm. To check that this critical point is a maximum, we find<br />

so<br />

V ww = −2h,<br />

V hh = −2w,<br />

V wh = 135 − 2w − 2h,<br />

D = V wwV hh − V 2 wh = 4hw − (135 − 2w − 2h) 2 .<br />

At w = h = 45, we have V ww = −2(45) < 0 and D = 4(45) 2 − (135 − 90 − 90) 2 = 6075 > 0, hence V is maximum<br />

at w = h = l = 45.<br />

Therefore, the suitcase with maximum volume is a cube with dimensions width = height = length = 45 cm.<br />

h<br />

w<br />

l<br />

Figure 15.12<br />

The box is shown in Figure 15.12. Cost of four sides = (2hl + 2wh)(1)c/. Cost of two bottoms = (2wl)(2)c/. Thus<br />

the total cost C (in cents) of the box is<br />

C = 2(hl + wh) + 4wl.<br />

But volume wlh = 512, so l = 512/(wh), thus<br />

C = 1024<br />

w<br />

To minimize C, find the critical points of C by solving<br />

We get<br />

+ 2wh +<br />

2048<br />

h .<br />

C h = 2w − 2048<br />

h 2 = 0,<br />

C w = 2h − 1024<br />

w 2 = 0.<br />

2wh 2 = 2048<br />

2hw 2 = 1024.


1056 Chapter Fifteen /SOLUTIONS<br />

Since w, h ≠ 0, we can divide the first equation by the second giving<br />

so<br />

thus<br />

2wh 2<br />

2hw 2 = 2048<br />

1024 ,<br />

h<br />

w = 2,<br />

h = 2w.<br />

Substituting this in C h = 0, we obtain h 3 = 2048, so h = 12.7 cm. Thus w = h/2 = 6.35 cm, and l = 512/(wh) =<br />

6.35 cm. Now we check that these dimensions minimize the cost C. We find that<br />

D = C hh C ww − Chw 2 = ( 4096 2048<br />

)(<br />

h3 w ) − 3 22 ,<br />

and at h = 12.7, w = 6.35, C hh > 0 and D = 16 − 4 > 0, thus C has a local minimum at h = 12.7 and w = 6.35.<br />

Since C increases without bound as w, h → 0 or ∞, this local minimum must be a global minimum.<br />

Therefore, the dimensions of the box that minimize the cost are w = 6.35 cm, l = 6.35 cm and h = 12.7 cm.<br />

28. The square of the distance from the point (x, y, z) to the origin is<br />

If the point is on the plane, z = 1 − 3x − 2y, we have<br />

At the critical point<br />

Simplifying gives<br />

S = x 2 + y 2 + z 2 .<br />

S = x 2 + y 2 + (1 − 3x − 2y) 2 .<br />

∂S<br />

= 2x + 2(1 − 3x − 2y)(−3) = 2(10x + 6y − 3) = 0<br />

∂x<br />

∂S<br />

= 2y + 2(1 − 3x − 2y)(−2) = 2(6x + 5y − 2) = 0.<br />

∂y<br />

10x + 6y = 3<br />

6x + 5y = 2,<br />

with solution x = 3/14, y = 1/7. At this point z = 1/14. We have<br />

D = S xxS yy − (S xy) 2 = (20)(10) − 12 2 = 56,<br />

so D > 0 and S xx > 0. Thus, the point x = 3/14, y = 1/7 is a local minimum. Since S → ∞ as x, y → ±∞, the local<br />

minimum is a global minimum. Thus, x = 3/14, y = 1/7, z = 1/14 is the closest point to the origin on the plane.<br />

29. We minimize the square of the distance from the point (x, y, z) to the origin:<br />

Since z 2 = 9 − xy − 3x, we have<br />

At a critical point<br />

so x = 2y, and<br />

S = x 2 + y 2 + z 2 .<br />

S = x 2 + y 2 + 9 − xy − 3x.<br />

∂S<br />

∂x = 2x − y − 3 = 0<br />

∂S<br />

= 2y − x = 0,<br />

∂y<br />

2(2y) − y − 3 = 0<br />

giving y = 1, so x = 2 and z 2 = 9 − 2 · 1 − 3 · 2 = 1, so z = ±1. We have<br />

D = S xxS yy − (S xy) 2 = 2 · 2 − (−1) 2 = 4 − 1 > 0,<br />

so, since D > 0 and S xx > 0, the critical points are local minima. Since S → ∞ as x, y → ±∞, the local minima are<br />

global minima.<br />

If x = 2, y = 1, z = ±1, we have S = 2 2 + 1 2 + 1 2 = 6, so the shortest distance to the origin is √ 6.


15.2 SOLUTIONS 1057<br />

30. (a) Let t be the number of years since 1960 and let P (t) be the population in millions in the year 1960 + t. We assume<br />

that P = Ce at , and therefore<br />

ln P = at + ln C.<br />

So, we plot ln P against t and find the line of best fit. Our data points are (0, ln 180), (10, ln 206), and (20, ln 226).<br />

Applying the method of least squares to find the best-fitting line, we find that<br />

Then, C = e 5.20 = 181.3 and so<br />

ln 226 − ln 180<br />

a = ≈ 0.0114,<br />

20<br />

ln 206 ln 226 5 ln 180<br />

ln C = − +<br />

3 6 6<br />

P (t) = 181.3e 0.0114t .<br />

In 1990, we have t = 30 and the predicted population in millions is<br />

P (30) = 181.3e 0.01141(30) = 255.3.<br />

≈ 5.20<br />

(b) The difference between the actual and the predicted population is about 6 million or 2 1 %. Given that only three data<br />

2<br />

points were used to calculate a and c, this discrepancy is not surprising. Thus, the 1990 census data does not mean<br />

that the assumption of exponential growth is unjustified.<br />

(c) In 2010, we have t = 50 and P (50) = 320.7.<br />

31. Let P (K, L) be the profit obtained using K units of capital and L units of labor. The cost of production is given by<br />

and the revenue function is given by<br />

Hence, the profit is<br />

C(K, L) = kK + lL,<br />

R(K, L) = pQ = pAK a L b .<br />

P = R − C = pAK a L b − (kK + lL).<br />

In order to find local maxima of P , we calculate the partial derivatives and see where they are zero. We have:<br />

∂P<br />

∂K = apAKa−1 L b − k,<br />

∂P<br />

∂L = bpAKa L b−1 − l.<br />

The critical points of the function P (K, L) are solutions (K, L) of the simultaneous equations:<br />

k<br />

a = pAKa−1 L b ,<br />

l<br />

b = pAKa L b−1 .<br />

Multiplying the first equation by K and the second by L, we get<br />

kK<br />

a = lL b ,<br />

and so<br />

K = la<br />

kb L.<br />

Substituting for K in the equation k/a = pAK a−1 L b , we get:<br />

We must therefore have<br />

Hence, if a + b ≠ 1,<br />

( )<br />

k la a−1<br />

a = pA L a−1 L b .<br />

kb<br />

( )<br />

L 1−a−b a a ( ) l a−1<br />

= pA<br />

.<br />

k b<br />

[ ( ) a a ( ) l (a−1)<br />

] 1/(1−a−b)<br />

L = pA<br />

,<br />

k b


1058 Chapter Fifteen /SOLUTIONS<br />

and<br />

[<br />

K = la<br />

kb L = la ( ) a a ( ) l (a−1)<br />

] 1/(1−a−b)<br />

pA<br />

.<br />

kb k b<br />

To see if this is really a local maximum, we apply the second derivative test. We have:<br />

Hence,<br />

∂ 2 P<br />

∂K 2 = a(a − 1)pAKa−2 L b ,<br />

∂ 2 P<br />

∂L 2 = b(b − 1)pAKa L b−2 ,<br />

∂ 2 P<br />

∂K∂L = abpAKa−1 L b−1 .<br />

( )<br />

D = ∂2 P ∂ 2 P ∂ 2 2<br />

∂K 2 ∂L − P<br />

2 ∂K∂L<br />

= ab(a − 1)(b − 1)p 2 A 2 K 2a−2 L 2b−2 − a 2 b 2 p 2 A 2 K 2a−2 L 2b−2<br />

= ab((a − 1)(b − 1) − ab)p 2 A 2 K 2a−2 L 2b−2<br />

= ab(1 − a − b)p 2 A 2 K 2a−2 L 2b−2 .<br />

Now a, b, p, A, K, and L are positive numbers. So, the sign of this last expression is determined by the sign of 1 − a − b.<br />

(a) We assumed that a + b < 1, so D > 0, and as 0 < a < 1, then ∂ 2 P/∂K 2 < 0 and so we have a unique local<br />

maximum. To verify that the local maximum is a global maximum, we focus on the cost. Let C = kK + lL. Since<br />

K ≥ 0 and L ≥ 0, K ≤ C/k and L ≤ C/l. Therefore the profit satisfies:<br />

P = pAK a L b − (kK + lL)<br />

( ) C a ( ) C b<br />

≤ pA<br />

− C<br />

k l<br />

= mC a+b − C<br />

where m = pA(1/k) a (1/l) b . Since a + b < 1, the profit is negative for large costs C, say C ≥ C 0 (C 0 = m 1−a−b<br />

will do). Therefore, in the KL-plane for K ≥ 0 and L ≥ 0, the profit is less than or equal to zero everywhere on or<br />

above the line kK + lL = C o. Thus the global maximum must occur inside the triangle bounded by this line and the<br />

K and L axes. Since P ≤ 0 on the K and L axes as well, the global maximum must be in the interior of the triangle<br />

at the unique local maximum we found.<br />

In the case a + b < 1, we have decreasing returns to scale. That is, if the amount of capital and labor used is<br />

multiplied by a constant λ > 0, we get less than λ times the production.<br />

(b) Now suppose a + b ≥ 1. If we multiply K and L by λ for some λ > 0, then<br />

32. We have<br />

We also see that<br />

So if a + b = 1, we have<br />

Q(λK, λL) = A(λK) a (λL) b = λ a+b Q(K, L).<br />

C(λK, λL) = λC(K, L).<br />

P (λK, λL) = λP (K, L).<br />

Thus, if λ = 2, so we are doubling the inputs K and L, then the profit P is doubled and hence there can be no<br />

maximum profit.<br />

If a + b > 1, we have increasing returns to scale and there can again be no maximum profit: doubling the inputs<br />

will more than double the profit. In this case, the profit increases without bound as K, L go toward infinity.<br />

f x = 2x(y + 1) 3 = 0<br />

only when x = 0 or y = −1<br />

f y = 3x 2 (y + 1) 2 + 2y = 0 never when y = −1 and only for y = 0 when x = 0<br />

We conclude that f x = 0 and f y = 0 only when x = 0, y = 0, so f has only one critical point, namely (0, 0).


15.2 SOLUTIONS 1059<br />

The second derivative test at (0, 0) gives<br />

D = f xxf yy − (f xy) 2 = 2(y + 1) 3 (6x 2 (y + 1) + 2) − (6x(y + 1) 2 ) 2<br />

= 2(1)(2) − 0 > 0 when x = 0, y = 0<br />

Since f xx > 0 at (0, 0), this means f has a local minimum at (0, 0).<br />

[Alternatively, if we expand (y + 1) 3 , then we can view f(x, y) as x 2 + y 2 + (terms of degree 3 or greater in x and<br />

y), which means that f behaves likes x 2 + y 2 near (0, 0).]<br />

Although (0, 0) is a local minimum, it cannot be a global minimum since for fixed x, say x = 1, the function f(x, y)<br />

is a cubic polynomial in y and cubics take on arbitrarily large positive and negative values.<br />

In the single-variable case, suppose a function f defined on the real line is differentiable and its derivative is continuous.<br />

Then if f has only one critical point, say x = 0, then if that critical point is a local minimum, it must also be a<br />

global minimum. This is because f ′ cannot change sign without f ′ = 0 so we must have f ′ < 0 for x < 0 and f ′ > 0<br />

for x > 0. Thus f is decreasing for all x < 0 and increasing for all x > 0, which makes x = 0 the global minimum for<br />

f.<br />

33. (a) We have f(2, 1) = 120.<br />

(i) If x > 20 then f(x, y) > 10x > 200 > f(2, 1).<br />

(ii) If y > 20 then f(x, y) > 20y > 400 > f(2, 1).<br />

(iii) If x < 0.01 and y ≤ 20 then f(x, y) > 80/(xy) > 80/((0.01)(20)) = 400 > f(2, 1).<br />

(iv) If y < 0.01 and x ≤ 20 then f(x, y) > 80/(xy) > 80/((20)(0.01)) = 400 > f(2, 1).<br />

(b) The continuous function f must achieve a minimum at some point (x 0, y 0) in the closed and bounded region R ′ :<br />

0.01 ≤ x ≤ 20, 0.01 ≤ y ≤ 20. Since (2, 1) is in R ′ , we must have f(x 0, y 0) ≤ f(2, 1). By part (a), f(x 0, y 0)<br />

is less than all values of f in the part of R that is outside R ′ , so f(x 0, y 0) is a minimum for f on all of R. Since<br />

(x 0, y 0) is not on the boundary of R, it must be a critical point of f.<br />

(c) The only critical point of f in R is the point (2, 1), so by part (b) f has a global minimum there.<br />

34. (a) The function f is continuous in the region R, but R is not closed and bounded so a special analysis is required.<br />

Notice that f(x, y) tends to ∞ as (x, y) tends farther and farther from the origin or tends toward any point on<br />

the x or y axis. This suggests that a minimum for f, if it exists, can not be too far from the origin or too close to the<br />

axes. For example, if x > 10 then f(x, y) > 4x > 40, and if y > 10 then f(x, y) > 5y > 50. If 0 < x < 0.1 then<br />

f(x, y) > 2/x > 20, and if 0 < y < 0.1 then f(x, y) > 3/y > 30.<br />

Since f(1, 1) = 14, a global minimum for f if it exists must be in the smaller region R ′ : 0.1 ≤ x ≤ 10,<br />

0.1 ≤ y ≤ 10. The region R ′ is closed and bounded and so f does have a minimum value at some point in R ′ , and<br />

since that value is at most 14, it is also a global minimum for all of R.<br />

(b) Since the region R has no boundary, the minimum value must occur at a critical point of f. At a critical point we<br />

have<br />

f x = − 2 x 2 + 4 = 0 fy = − 3 y 2 + 5 = 0.<br />

The only critical point is ( √ 1/2, √ 3/5) ≈ (0.7071, 0.7746), at which f achieves the minimum value<br />

f( √ 1/2, √ 3/5) = 4 √ 2 + 2 √ 15 ≈ 13.403.<br />

35. (a) By the chain rule applied to M(a) = f(h(a), a) we have M ′ (a) = f x(h(a), a)h ′ (a) + f a(h(a), a)da/da =<br />

f x(h(a), a)h ′ (a) + f a(h(a), a). Since x = h(a) is a critical point of g(x) = f(x, a), we have g ′ (h(a)) =<br />

f x(h(a), a) = 0. Thus M ′ (a) = f a(h(a), a).<br />

(b) Let n(a) = f(x, a) be a cross-section of f with x fixed. We have n ′ (a) = f a(x, a). If x = h(a), then n(a) =<br />

f(h(a), a) = M(a) so the point (a, M(a)) is on the graph of n. And n ′ (a) = f a(h(a), a) = M ′ (a) so n and M<br />

have the same slope at the point (a, h(a)). Their graphs are tangent.<br />

(c) The quadratic function g(x) = f(x, a) = −(1/2)x 2 + 2ax − a 2 has a maximum where g ′ (x) = −x + 2a = 0 or<br />

x = 2a. Thus h(a) = 2a. The maximum value is M(a) = g(h(a)) = f(2a, a) = a 2 . The graph in Figure 15.13<br />

shows that the graph of M(a) is the upper boundary, called the upper envelope, of the family of cross-sections of f<br />

with x fixed.


1060 Chapter Fifteen /SOLUTIONS<br />

y<br />

M(a)<br />

f(3, a)<br />

x<br />

f(−2, a)<br />

f(−0, a) f(1, a)<br />

f(2, a) f(−1, a)<br />

Figure 15.13<br />

Solutions for Section 15.3<br />

Exercises<br />

1. Our objective function is f(x, y) = x + y and our equation of constraint is g(x, y) = x 2 + y 2 = 1. To optimize f(x, y)<br />

with Lagrange multipliers, we solve ∇f(x, y) = λ∇g(x, y) subject to g(x, y) = 1. The gradients of f and g are<br />

So the equation ∇f = λ∇g becomes<br />

Solving for λ gives<br />

∇f(x, y) = ⃗i + ⃗j ,<br />

∇g(x, y) = 2x⃗i + 2y⃗j .<br />

⃗i + ⃗j = λ(2x⃗i + 2y⃗j )<br />

λ = 1<br />

2x = 1<br />

2y ,<br />

which tells us that x = y. Going back to our equation of constraint, we use the substitution x = y to solve for y:<br />

g(y, y) = y 2 + y 2 = 1<br />

2y 2 = 1<br />

y 2 = 1 2<br />

√ √<br />

1 2<br />

y = ±<br />

2 = ± 2 .<br />

Since x = y, our critical points are ( √ 2<br />

, √ 2<br />

) and (− √ 2<br />

, − √ 2<br />

). Since the constraint is closed and bounded, maximum<br />

2 2 2 2<br />

and minimum values of f subject to the constraint exist. Evaluating f at the critical points we find that the maximum<br />

value is f( √ 2<br />

, √ 2<br />

) = √ 2 and the minimum value is f(− √ 2<br />

, − √ 2<br />

) = −√ 2.<br />

2 2 2 2<br />

2. Our objective function is f(x, y) = 3x − 2y and our equation of constraint is g(x, y) = x 2 + 2y 2 = 44. Their gradients<br />

are<br />

∇f(x, y) = 3⃗i − 2⃗j ,<br />

∇g(x, y) = 2x⃗i + 4y⃗j .


15.3 SOLUTIONS 1061<br />

So the equation ∇f = λ∇g becomes 3⃗i − 2⃗j = λ(2x⃗i + 4y⃗j ). Solving for λ gives us<br />

which we can use to find x in terms of y:<br />

λ = 3<br />

2x = −2<br />

4y ,<br />

3<br />

2x = −2<br />

4y<br />

−4x = 12y<br />

x = −3y.<br />

Using this relation in our equation of constraint, we can solve for y:<br />

x 2 + 2y 2 = 44<br />

(−3y) 2 + 2y 2 = 44<br />

9y 2 + 2y 2 = 44<br />

11y 2 = 44<br />

y 2 = 4<br />

y = ±2.<br />

Thus, the critical points are (−6, 2) and (6, −2). Since the constraint is closed and bounded, maximum and minimum<br />

values of f subject to the constraint exist. Evaluating f at the critical points, we find that the maximum is f(6, −2) =<br />

18 + 4 = 22 and the minimum value is f(−6, 2) = −18 − 4 = −22.<br />

3. Our objective function is f(x, y) = xy and our equation of constraint is g(x, y) = 4x 2 + y 2 = 8. Their gradients are<br />

∇f(x, y) = y⃗i + x⃗j ,<br />

∇g(x, y) = 8x⃗i + 2y⃗j .<br />

So the equation ∇f = λ∇g becomes y⃗i + x⃗j = λ(8x⃗i + 2y⃗j ). This gives<br />

Multiplying, we get<br />

8xλ = y and 2yλ = x.<br />

8x 2 λ = 2y 2 λ.<br />

If λ = 0, then x = y = 0, which does not satisfy the constraint equation. So λ ≠ 0 and we get<br />

To find x, we substitute for y in our equation of constraint.<br />

2y 2 = 8x 2<br />

y 2 = 4x 2<br />

y = ±2x.<br />

4x 2 + y 2 = 8<br />

4x 2 + 4x 2 = 8<br />

x 2 = 1<br />

x = ±1<br />

So our critical points are (1, 2), (1, −2), (−1, 2) and (−1, −2). Since the constraint is closed and bounded, maximum<br />

and minimum values of f subject to the constraint exist. Evaluating f(x, y) at the critical points, we have<br />

f(1, 2) = f(−1, −2) = 2<br />

f(1, −2) = f(1, −2) = −2.<br />

Thus, the maximum value of f on g(x, y) = 8 is 2, and the minimum value is −2.


1062 Chapter Fifteen /SOLUTIONS<br />

4. The objective function is f(x, y) = x 2 + y 2 and the equation of constraint is g(x, y) = x 4 + y 4 = 2. Their gradients are<br />

∇f(x, y) = 2x⃗i + 2y⃗j ,<br />

∇g(x, y) = 4x 3 ⃗i + 4y 3 ⃗j .<br />

So the equation ∇f = λ∇g becomes 2x⃗i + 2y⃗j = λ(4x 3 ⃗i + 4y 3 ⃗j ). This tells us that<br />

2x = 4λx 3 ,<br />

2y = 4λy 3 .<br />

Now if x = 0, the first equation is true for any value of λ. In particular, we can choose λ which satisfies the second<br />

equation. Similarly, y = 0 is solution.<br />

Assuming both x ≠ 0 and y ≠ 0, we can divide to solve for λ and find<br />

Going back to our equation of constraint, we find<br />

λ = 2x<br />

4x = 2y<br />

3 4y 3<br />

1<br />

2x = 1<br />

2 2y 2<br />

y 2 = x 2<br />

y = ±x.<br />

g(0, y) = 0 4 + y 4 = 2, so y = ± 4√ 2<br />

g(x, 0) = x 4 + 0 4 = 2, so x = ± 4√ 2<br />

g(x, ±x) = x 4 + (±x) 4 = 2, so x = ±1.<br />

Thus, the critical points are (0, ± 4√ 2), (± 4√ 2, 0), (1, ±1) and (−1, ±1). Since the constraint is closed and bounded,<br />

maximum and minimum values of f subject to the constraint exist. Evaluating f at the critical points, we find<br />

f(1, 1) = f(1, −1) = f(−1, 1) = f(−1, −1) = 2,<br />

f(0, 4√ 2) = f(0, − 4√ 2) = f( 4√ 2, 0) = f(− 4√ 2, 0) = √ 2.<br />

Thus, the minimum value of f(x, y) on g(x, y) = 2 is √ 2 and the maximum value is 2.<br />

5. The objective function is f(x, y) = x 2 + y 2 and the constraint equation is g(x, y) = 4x − 2y = 15, so grad f =<br />

(2x)⃗i + (2y)⃗j and grad g = 4⃗i − 2⃗j . Setting grad f = λ grad g gives<br />

2x = 4λ,<br />

2y = −2λ.<br />

From the first equation we have λ = x/2, and from the second equation we have λ = −y. Setting these equal gives<br />

y = −0.5x.<br />

Substituting this into the constraint equation 4x − 2y = 15 gives x = 3. The only critical point is (3, −1.5).<br />

We have f(3, −1.5) = (3) 2 + (1.5) 2 = 11.25. One way to determine if this point gives a maximum or minimum<br />

value or neither for the given constraint is to examine the contour diagram of f with the constraint sketched in, Figure<br />

15.14. It appears that moving away from the point P = (3, −1.5) in either direction along the constraint increases the<br />

value of f, so (3, −1.5) is a point of minimum value.


y<br />

1<br />

−1<br />

−2<br />

−3<br />

Constraint: 4x − 2y = 15<br />

5 10 15 20<br />

x<br />

1 2 3 4<br />

P = (3, −1.5)<br />

15.3 SOLUTIONS 1063<br />

y<br />

2<br />

x 2 − y 2 = 1<br />

2.25<br />

1.5<br />

0.75<br />

x<br />

−2 2<br />

(− √ 0<br />

5/2, −1/2)<br />

( √ 5/2, 1/2)<br />

Figure 15.14<br />

−2<br />

Figure 15.15: Graph of x 2 − y 2 = 1<br />

6. Our objective function is f(x, y) = x 2 + y and our equation of constraint is g(x, y) = x 2 − y 2 = 1. Their gradients are<br />

Thus ∇f = λ∇g gives<br />

∇f(x, y) = 2x⃗i + ⃗j ,<br />

∇g(x, y) = 2x⃗i − 2y⃗j .<br />

2x = λ2x<br />

1 = −λ2y<br />

But x cannot be zero, since the constraint equation, −y 2 = 1, would then have no real solution for y. So the equation<br />

∇f = λ∇g becomes<br />

Substituting this into our equation of constraint we find<br />

g(x, − 1 2 ) = x2 −<br />

λ = 2x<br />

2x = 1<br />

−2y<br />

1 = 1<br />

−2y<br />

−2y = 1<br />

y = − 1 2 .<br />

(<br />

− 1 2) 2<br />

= 1<br />

x 2 = 5 4<br />

x = ±<br />

So the critical points are ( √ 5<br />

, − 1 ) and (− √ 5<br />

, − 1 ). Evaluating f at these points we find f( √ 5<br />

, − 1 ) = f(− √ 5<br />

, − 1 ) =<br />

2 2 2 2 2 2 2 2<br />

5<br />

− 1 = 3 . This is the minimum value for f(x, y) constrained to g(x, y) = 1. To see this, note that for 4 2 4 x2 = y 2 + 1,<br />

f(x, y) = y 2 + 1 + y = (y + 1/2) 2 + 3/4 ≥ 3/4. Alternatively, see Figure 15.15. To see that f has no maximum on<br />

g(x, y) = 1, note that f → ∞ as x → ∞ and y → ∞ on the part of the graph of g(x, y) = 1 in quadrant I.<br />

7. The objective function is f(x, y) = x 2 − xy + y 2 and the equation of constraint is g(x, y) = x 2 − y 2 = 1. The gradients<br />

of f and g are<br />

√<br />

5<br />

2 .<br />

∇f(x, y) = (2x − y)⃗i + (−x + 2y)⃗j ,<br />

∇g(x, y) = 2x⃗i − 2y⃗j .<br />

Therefore the equation ∇f(x, y) = λ∇g(x, y) gives<br />

2x − y = 2λx<br />

−x + 2y = −2λy<br />

x 2 − y 2 = 1.


1064 Chapter Fifteen /SOLUTIONS<br />

Let us suppose that λ = 0. Then 2x = y and 2y = x give x = y = 0. But (0, 0) is not a solution of the third equation,<br />

so we conclude that λ ≠ 0. Now let’s multiply the first two equations<br />

−2λy(2x − y) = 2λx(−x + 2y).<br />

As λ ≠ 0, we can cancel it in the equation above and after doing the algebra we get<br />

which gives x = (2 + √ 3)y or x = (2 − √ 3)y.<br />

If x = (2 + √ 3)y, the third equation gives<br />

x 2 − 4xy + y 2 = 0<br />

(2 + √ 3) 2 y 2 − y 2 = 1<br />

so y ≈ ±0.278 and x ≈ ±1.038. These give the critical points (1.038, 0.278), (−1.038, −0.278).<br />

If x = (2 − √ 3)y, from the third equation we get<br />

(2 − √ 3) 2 y 2 − y 2 = 1.<br />

But (2 − √ 3) 2 − 1 ≈ −0.928 < 0 so the equation has no solution. Evaluating f gives<br />

Since y → ∞ on the constraint, rewriting f as<br />

f(1.038, 0.278) = f(−1.038, −0.278) ≈ 0.866<br />

f(x, y) =<br />

(<br />

x − y ) 2<br />

3 +<br />

2 4 y2<br />

shows that f has no maximum on the constraint. The minimum value of f is 0.866. See Figure 15.16.<br />

y<br />

x 2 − y 2 = 1<br />

−2<br />

(−1.038, −0.278) 0.2<br />

0.866<br />

(1.038, 0.278)<br />

x<br />

2<br />

2<br />

Figure 15.16<br />

8. The objective function is f(x, y, z) = x + 3y + 5z and the equation of constraint is g(x, y, z) = x 2 + y 2 + z 2 = 1.<br />

Their gradients are<br />

∇f(x, y, z) = ⃗i + 3⃗j + 5 ⃗ k ,<br />

∇g(x, y, z) = 2x⃗i + 2y⃗j + 2z ⃗ k .<br />

So the equation ∇f = λ∇g becomes⃗i + 3⃗j + 5 ⃗ k = λ(2x⃗i + 2y⃗j + 2z ⃗ k ). Solving for λ we find<br />

Which provides us with the equations<br />

λ = 1<br />

2x = 3<br />

2y = 5<br />

2z .<br />

2y = 6x<br />

10x = 2z.


15.3 SOLUTIONS 1065<br />

Solving the first equation for y gives us y = 3x. Solving the second equation for z gives us z = 5x. Substituting these<br />

into the equation of constraint, we can find x:<br />

x 2 + (3x) 2 + (5x) 2 = 1<br />

x 2 + 9x 2 + 25x 2 = 1<br />

35x 2 = 1<br />

x 2 = 1<br />

35<br />

x = ±<br />

√<br />

√<br />

1 35<br />

35 = ± 35 .<br />

Since y = 3x and z = 5x, the critical points are at ±( √ 35<br />

, 3 √ 35<br />

, √ 35<br />

). Since the constraint is closed and bounded, maximum<br />

and minimum values of f subject to the constraint exist. Evaluating f at the critical points, we find the maximum<br />

35 35 7<br />

is f( √ 35<br />

, 3 √ 35<br />

, √ 35<br />

) = √ 35 35 = √ 35, and the minimum value is f(− √ 35<br />

, −3 √ 35<br />

, − √ 35<br />

) = −√ 35.<br />

35 35 7 35 35 35 7<br />

9. Our objective function is f(x, y, z) = x 2 −2y +2z 2 and our equation of constraint is g(x, y, z) = x 2 +y 2 +z 2 −1 = 0.<br />

To optimize f(x, y, z) with Lagrange multipliers, we solve ∇f(x, y, z) = λ∇g(x, y, z) subject to g(x, y, z) = 0. The<br />

gradients of f and g are<br />

We get,<br />

From the first equation we get x = 0 or λ = 1.<br />

If x = 0 we have<br />

∇f(x, y, z) = 2x⃗i − 2⃗j + 4z ⃗ k ,<br />

∇g(x, y) = 2x⃗i + 2y⃗j + 2z ⃗ k .<br />

x = λx<br />

−1 = λy<br />

2z = λz<br />

x 2 + y 2 + z 2 = 1.<br />

−1 = λy<br />

2z = λz<br />

y 2 + z 2 = 1.<br />

From the second equation z = 0 or λ = 2. So if z = 0, we have y = ±1 and we get the solutions (0, 1, 0),(0, −1, 0). If<br />

z ≠ 0 then λ = 2 and y = − 1 . So 2 z2 = 3 which gives the solutions (0, − 1 , √ 3<br />

), (0, − 1 , − √ 3<br />

).<br />

4 2 2 2<br />

If x ≠ 0, then λ = 1, so y = −1, which implies, from the equation x 2 + y 2 + z 2 = 1, that x = 0, which contradicts<br />

the assumption.<br />

Since the constraint is closed and bounded, maximum and minimum values of f subject to the constraint exist. Therefore,<br />

evaluating f at the critical points, we get f(0, 1, 0) = −2, f(0, −1, 0) = 2 and f(0, − 1 , √ 3<br />

) = f(0, − 1 , − √ 3<br />

) =<br />

2 2 2 2<br />

4. So the maximum value of f is 4 and the minimum is −2.<br />

10. Our objective function is f(x, y, z) = 2x + y + 4z and our equation of constraint is g(x, y, z) = x 2 + y + z 2 = 16.<br />

Their gradients are<br />

∇f(x, y, z) = 2⃗i + 1⃗j + 4 ⃗ k ,<br />

∇g(x, y, z) = 2x⃗i + 1⃗j + 2z ⃗ k .<br />

So the equation ∇f = λ∇g becomes 2⃗i + 1⃗j + 4 ⃗ k = λ(2x⃗i + 1⃗j + 2z ⃗ k ). Solving for λ we find<br />

λ = 2<br />

2x = 1 1 = 4<br />

2z<br />

λ = 1 x = 1 = 2 z .<br />

Which tells us that x = 1 and z = 2. Going back to our equation of constraint, we can solve for y.<br />

g(1, y, 2) = 16<br />

1 2 + y + 2 2 = 16<br />

y = 11.


1066 Chapter Fifteen /SOLUTIONS<br />

So our one critical point is at (1, 11, 2). The value of f at this point is f(1, 11, 2) = 2 + 11 + 8 = 21. This is the<br />

maximum value of f(x, y, z) on g(x, y, z) = 16. To see this, note that for y = 16 − x 2 − z 2 ,<br />

f(x, y, z) = 2x + 16 − x 2 − z 2 + 4z = 21 − (x − 1) 2 − (z − 2) 2 ≤ 21.<br />

As y → −∞, the point (− √ 16 − y, y, 0) is on the constraint and f(− √ 16 − y, y, 0) → −∞, so there is no minimum<br />

value for f(x, y, z) on g(x, y, z) = 16.<br />

11. The region x 2 + y 2 ≤ 4 is the shaded disk of radius 2 centered at the origin (including the circle x 2 + y 2 = 4) shown in<br />

Figure 15.17.<br />

We will first find the local maxima and minima in the interior of the disk. So we need to find the extrema of<br />

For this we compute the critical points:<br />

f(x, y) = x 2 + 2y 2 in the region x 2 + y 2 < 4.<br />

f x = 2x = 0<br />

f y = 4y = 0<br />

So the critical point is (0, 0). As f xx(0, 0) = 2, f yy(0, 0) = 4 and f xy(0, 0) = 0 we have<br />

D = f xx(0, 0) · f yy(0, 0) − (f xy(0, 0)) 2 = 8 > 0 and f xx(0, 0) = 2 > 0.<br />

Therefore (0, 0) is a minimum point and f(0, 0) = 0.<br />

Now let’s find the local extrema of f on the boundary of the disk, hence this time we have to solve a constraint<br />

problem. We want the extrema of f(x, y) = x 2 + 2y 2 subject to g(x, y) = x 2 + y 2 − 4 = 0. We use Lagrange<br />

multipliers:<br />

grad f = λ grad g and x 2 + y 2 = 4,<br />

which give<br />

2x = 2λx<br />

4y = 2λy<br />

x 2 + y 2 = 4.<br />

From the first equation we have x = 0 or λ = 1. If x = 0, from the last equation y 2 = 4 and therefore (0, 2) and<br />

(0, −2) are solutions.<br />

If x ≠ 0 then λ = 1 and from the second equation y = 0. Substituting this into the third equation we get x 2 = 4 so<br />

(2, 0) and (−2, 0) are the other two solutions.<br />

The region x 2 + y 2 ≤ 4 is closed and bounded, so maximum and minimum values of f in the the region exist.<br />

Therefore, as f(0, 2) = f(0, −2) = 8 and f(2, 0) = f(−2, 0) = 4, (0, 2) and (0, −2) are global maxima and (0, 0) is<br />

the global minimum on the whole region. The maximum value of f is 8 and the minimum value of f is 0.<br />

y<br />

y<br />

4<br />

16<br />

8<br />

1 4<br />

−4 4<br />

x<br />

− √ 2<br />

√<br />

2<br />

1<br />

−1<br />

3<br />

5<br />

x<br />

√<br />

2<br />

−3<br />

−4<br />

Figure 15.17<br />

−5<br />

− √ 2<br />

Figure 15.18


15.3 SOLUTIONS 1067<br />

12. The region x 2 + y 2 ≤ 2 is the shaded disk of radius √ 2 centered at the origin (including the circle x 2 + y 2 = 2) as shown<br />

in Figure 15.18.<br />

We first find the local maxima and minima of f in the interior of our disk. So we need to find the extrema of<br />

As<br />

f(x, y) = x + 3y, in the region x 2 + y 2 < 2.<br />

f x = 1<br />

f y = 3<br />

f does not have critical points. Now let’s find the local extrema of f on the boundary of the disk. We want to find the<br />

extrema of f(x, y) = x + 3y subject to the constraint g(x, y) = x 2 + y 2 − 2 = 0. We use Lagrange multipliers<br />

which give<br />

grad f = λ grad g and x 2 + y 2 = 2,<br />

1 = 2λx<br />

3 = 2λy<br />

x 2 + y 2 = 2.<br />

As λ cannot be zero, we solve for x and y in the first two equations and get x = 1 and y = 3 . Plugging into the third<br />

2λ 2λ<br />

equation gives<br />

8λ 2 = 10<br />

so λ = ± √ 5<br />

2<br />

1<br />

and we get the solutions ( √<br />

3<br />

5<br />

, √<br />

5<br />

) and (− √ 1<br />

5<br />

, − √ 3<br />

5<br />

). Evaluating f at these points gives<br />

f( √ 1 3<br />

, √ ) = 2 √ 5<br />

5 5<br />

f(− 1 √<br />

5<br />

, − 3 √<br />

5<br />

) = −2 √ 5<br />

and<br />

The region x 2 + y 2 ≤ 2 is closed and bounded, so maximum and minimum values of f in the region exist. Therefore<br />

( √ 1 3<br />

5<br />

, √<br />

5<br />

) is a global maximum of f and (− √ 1<br />

5<br />

, − √ 3<br />

5<br />

) is a global minimum of f on the whole region x 2 + y 2 ≤ 2.<br />

13. The domain x 2 +2y 2 ≤ 1 is the shaded interior of the ellipse x 2 +2y 2 = 1 including the boundary, shown in Figure 15.19.<br />

y<br />

− √ 1<br />

2<br />

−0.5<br />

−0.3<br />

−0.1 0.1<br />

0.5<br />

0.3<br />

−1 1<br />

x<br />

− 1 √<br />

2<br />

Figure 15.19<br />

First we want to find the local maxima and minima of f in the interior of the ellipse. So we need to find the extrema<br />

of<br />

f(x, y) = xy, in the region x 2 + 2y 2 < 1.<br />

For this we compute the critical points:<br />

f x = y = 0 and f y = x = 0.<br />

So there is one critical point, (0, 0). As f xx(0, 0) = 0, f yy(0, 0) = 0 and f xy(0, 0) = 1 we have<br />

D = f xx(0, 0) · f yy(0, 0) − (f xy(0, 0)) 2 = −1 < 0


1068 Chapter Fifteen /SOLUTIONS<br />

so (0, 0) is a saddle and f does not have local extrema in the interior of the ellipse.<br />

Now let’s find the local extrema of f on the boundary, hence this time we’ll have a constraint problem. We want the<br />

extrema of f(x, y) = xy subject to g(x, y) = x 2 + 2y 2 − 1 = 0. We use Lagrange multipliers:<br />

which give<br />

From the first two equations we get<br />

grad f = λ grad g and x 2 + 2y 2 = 1<br />

y = 2λx<br />

x = 4λy<br />

x 2 + 2y 2 = 1<br />

xy = 8λ 2 xy.<br />

So x = 0 or y = 0 or 8λ 2 = 1.<br />

If x = 0, from the last equation 2y 2 = 1 so y = ± √ 2<br />

and we get the solutions (0, √ 2<br />

) and (0, − √ 2<br />

).<br />

2 2 2<br />

If y = 0, from the last equation we get x 2 = 1 and so the solutions are (1, 0) and (−1, 0).<br />

If x ≠ 0 and y ≠ 0 then 8λ 2 = 1, hence λ = ± 1<br />

2 √ . For λ = 1<br />

2 2 √ 2<br />

x = √ 2y<br />

and plugging into the third equation gives 4y 2 = 1 so we get the solutions ( √ 2<br />

2 , 1 2 ) and (− √ 2<br />

2 , − 1 2 ).<br />

For λ = − 1<br />

2 √ 2 we get x = − √ 2y<br />

and plugging into the third equation gives 4y 2 = 1, and the solutions ( √ 2<br />

, − 1 ) and (− √ 2<br />

, 1 ). So finally we have the<br />

2 2 2 2<br />

solutions: (1, 0), (−1, 0), ( √ 2<br />

, 1 ), (− √ 2<br />

, − 1 ), ( √ 2<br />

, − 1 ), (− √ 2<br />

, 1 ).<br />

2 2 2 2 2 2 2 2<br />

Evaluating f at these points gives:<br />

√ √<br />

2<br />

2<br />

f(0,<br />

2 ) = f(0, − ) = f(1, 0) = f(−1, 0) = 0<br />

2<br />

√<br />

2<br />

f(<br />

2 , 1 √<br />

2<br />

2 ) = f(− 2 , − 1 √<br />

2<br />

2 ) = 4<br />

√<br />

2<br />

f(<br />

2 , − 1 √<br />

2<br />

2 ) = f(− 2 , 1 √<br />

2<br />

2 ) = − 4 .<br />

The region x 2 + 2y 2 ≤ 1 is closed and bounded, so the maximum and minimum values of f in the region exist. Hence<br />

the maximum value of f is √ 2<br />

and the minimum value of f is − √ 2<br />

.<br />

4 4<br />

14. The region x 2 + y 2 ≤ 1 is the shaded disk of radius 1 centered at the origin (including the circle x 2 + y 2 = 1) shown in<br />

Figure 15.20.<br />

Let’s first compute the critical points of f in the interior of the disk. We have<br />

f x = 3x 2 = 0<br />

f y = −2y = 0,<br />

whose solution is x = y = 0. So the only one critical point is (0, 0). As f xx(0, 0) = 0, f yy(0, 0) = −2 and f xy(0, 0) =<br />

0,<br />

D = f xx(0, 0) · f yy(0, 0) − (f xy(0, 0)) 2 = 0<br />

which does not tell us anything about the nature of the critical point (0, 0).<br />

But, if we choose x,y very small in absolute value and such that x 3 > y 2 , then f(x, y) > 0. If we choose x,y very<br />

small in absolute value and such that x 3 < y 2 , then f(x, y) < 0. As f(0, 0) = 0, we conclude that (0, 0) is a saddle<br />

point.<br />

We can get the same conclusion looking at the level curves of f around (0, 0), as shown in Figure 15.21.<br />

So, f does not have extrema in the interior of the disk.<br />

Now, let’s find the local extrema of f on the circle x 2 + y 2 = 1. So we want the extrema of f(x, y) = x 3 − y 2<br />

subject to the constraint g(x, y) = x 2 + y 2 − 1 = 0. Using Lagrange multipliers we get<br />

grad f = λ grad g and x 2 + y 2 = 1,


15.3 SOLUTIONS 1069<br />

which gives<br />

3x 2 = 2λx<br />

−2y = 2λy<br />

x 2 + y 2 = 1.<br />

From the second equation y = 0 or λ = −1.<br />

If y = 0, from the third equation we get x 2 = 1, which gives the solutions (1, 0), (−1, 0).<br />

If y ≠ 0 then λ = −1 and from the first equation we get 3x 2 = −2x, hence x = 0 or x = − 2 . If x = 0, from the<br />

3<br />

third equation we get y 2 = 1, so the solutions (0, 1),(0, −1). If x = − 2 , from the third equation we get 3 y2 = 5 , so the<br />

9<br />

solutions (− 2 , √ 5<br />

), (− 2 , − √ 5<br />

).<br />

3 3 3 3<br />

Evaluating f at these points we get<br />

f(1, 0) = 1,<br />

f(−1, 0) = f(0, 1) = f(0, −1) = −1<br />

and<br />

f<br />

(<br />

− 2 3 , − √<br />

5<br />

3<br />

) (<br />

= f − 2 √ )<br />

5<br />

3 , 3<br />

= − 23<br />

27 .<br />

The region x 2 + y 2 ≤ 1 is closed and bounded, so maximum and minimum values of f in the region exist. Therefore<br />

the maximum value of f is 1 and the minimum value is −1.<br />

1<br />

y<br />

1<br />

y<br />

x<br />

1<br />

−0.7<br />

−0.3<br />

−0.1<br />

0 0.3 0.7<br />

−1 1<br />

x<br />

−1<br />

Figure 15.20<br />

Figure 15.21: Level curves of f<br />

15. The region x + y ≥ 1 is the shaded half plane (including the line x + y = 1) shown in Figure 15.22.<br />

y<br />

3<br />

2<br />

1<br />

−1<br />

−3<br />

−5<br />

1<br />

3 5<br />

1 2 3<br />

x<br />

Figure 15.22<br />

Let’s look for the critical points of f in the interior of the region. As<br />

f x = 3x 2<br />

f y = 1


1070 Chapter Fifteen /SOLUTIONS<br />

there are no critical points inside the shaded region. Now let’s find the extrema of f on the boundary of our region. We<br />

want the extrema of f(x, y) = x 3 + y subject to the constraint g(x, y) = x + y − 1 = 0. We use Lagrange multipliers<br />

which give<br />

grad f = λ grad g and x + y = 1,<br />

3x 2 = λ<br />

1 = λ<br />

x + y = 1.<br />

From the first two equations we get 3x 2 = 1, so the solutions are<br />

Evaluating f at these points we get<br />

( 1 √<br />

3<br />

, 1 − 1 √<br />

3<br />

) and (− 1 √<br />

3<br />

, 1 + 1 √<br />

3<br />

).<br />

f( √ 1 , 1 − √ 1 ) = 1 − 2<br />

3 3 3 √ 3<br />

f(− √ 1 , 1 + √ 1 ) = 1 + 2<br />

3 3 3 √ 3 .<br />

From the contour diagram in Figure 15.22, we see that ( 1 √<br />

3<br />

, 1 − 1 √<br />

3<br />

) is a local minimum and (− 1 √<br />

3<br />

, 1 + 1 √<br />

3<br />

) is a local<br />

maximum of f on x + y = 1. Are they global extrema as well?<br />

If we take x very big and y = 1 − x then f(x, y) = x 3 + y = x 3 − x + 1 which can be made as big as we want (if<br />

we choose x big enough). So there will be no global maximum.<br />

Similarly, taking x negative with big absolute value and y = 1 − x, f(x, y) = x 3 + y = x 3 − x + 1 can be made as<br />

small as we want (if we choose x small enough). So there is no global minimum. This can also be seen from Figure 15.22.<br />

Problems<br />

16. We know that a maximum or minimum value of f subject to the constraint equation g(x, y) = c occurs where grad f<br />

is parallel to grad g, or at the endpoints of the constraint. The vectors grad f and grad g are parallel where the graph of<br />

g(x, y) = c is tangent to the contours of f, which occurs at approximately x = 6 and y = 6. At the point (6, 6), we<br />

have f = 400. The graph of g(x, y) = c crosses the contours f = 300, f = 200, f = 100 but does not cross any<br />

contours with f-values greater than 400. We see that the maximum of f subject to the constraint is 400 at the point (6, 6).<br />

It appears that f takes on its minimum value (less than 100) at one of the endpoints, which are approximately (10.5, 0)<br />

and (0, 13.5).<br />

17. (a) The contour for z = 1 is the line 1 = 2x + y, or y = −2x + 1. The contour for z = 3 is the line 3 = 2x + y, or<br />

y = −2x + 3. The contours are all lines with slope −2. See Figure 15.23.<br />

y<br />

y<br />

5<br />

4<br />

3<br />

−7<br />

2<br />

−5<br />

−3 1<br />

−1<br />

1<br />

−5 −4 −3 −2 −1 1 2 3 4 5<br />

−1 3<br />

5<br />

−2<br />

7<br />

−3<br />

−4<br />

−5<br />

x<br />

5<br />

4<br />

3<br />

−7<br />

2<br />

−5<br />

−3 1<br />

−1<br />

1<br />

−5 −4 −3 −2 −1 1 2 3 4 5<br />

−1 3<br />

5<br />

−2<br />

7<br />

−3<br />

−4<br />

−5<br />

x<br />

Figure 15.23<br />

Figure 15.24


15.3 SOLUTIONS 1071<br />

(b) The graph of x 2 + y 2 = 5 is a circle of radius √ 5 = 2.236 centered at the origin. See Figure 15.24.<br />

(c) The circle representing the constraint equation in Figure 15.24 appears to be tangent to the contour close to z = 5 at<br />

the point (2, 1), and this is the contour with the highest z-value that the circle intersects. The circle is tangent to the<br />

contour z = −5 approximately at the point (−2, −1), and this is the contour with the lowest z-value that the circle<br />

intersects. Therefore, subject to the constraint x 2 + y 2 = 5, the function f has a maximum value of about 5 at the<br />

point (2, 1) and a minimum value of about −5 at the point (−2, −1).<br />

Since the radius vector, 2⃗i + ⃗j , at the point (2, 1) is perpendicular to the line 2x + y = 5, the maximum is<br />

exactly 5 and occurs at (2, 1). Similarly, the minimum is exactly −5 and occurs at (−2, −1).<br />

(d) The objective function is f(x, y) = 2x + y and the constraint equation is g(x, y) = x 2 + y 2 = 5, and so grad f =<br />

2⃗i + ⃗j and grad g = (2x)⃗i + (2y)⃗j . Setting grad f = λ grad g gives<br />

2 = λ(2x),<br />

1 = λ(2y).<br />

On the constraint, x ≠ 0 and y ≠ 0. Thus, from the first equation, we have λ = 1/x, and from the second equation<br />

we have λ = 1/(2y). Setting these equal gives<br />

x = 2y.<br />

Substituting this into the constraint equation x 2 +y 2 = 5 gives (2y) 2 +y 2 = 5 so y = −1 and y = 1. Since x = 2y,<br />

the maximum or minimum values occur at (2, 1) or (−2, −1). Since f(2, 1) = 5 and f(−2, −1) = −5, the function<br />

f(x, y) = 2x + y subject to the constraint x 2 + y 2 = 5 has a maximum value of 5 at the point (2, 1) and a minimum<br />

value of −5 at the point (−2, −1). This confirms algebraically what we observed graphically in part (c).<br />

18. We want to minimize<br />

C = f(q 1, q 2) = 2q 2 1 + q 1q 2 + q 2 2 + 500<br />

subject to the constraint q 1 + q 2 = 200 or g(q 1, q 2) = q 1 + q 2 − 200 = 0.<br />

Since ∇f = (4q 1 + q 2)⃗i + (2q 2 + q 1)⃗j and ∇g = ⃗i + ⃗j , ∇f = λ∇g gives<br />

4q 1 + q 2 = λ<br />

2q 2 + q 1 = λ.<br />

Solving we get<br />

so<br />

We want<br />

Therefore<br />

4q 1 + q 2 = 2q 2 + q 1<br />

3q 1 = q 2.<br />

q 1 + q 2 = 200<br />

q 1 + 3q 1 = 4q 1 = 200.<br />

q 1 = 50 units, q 2 = 150 units.<br />

19.<br />

✻<br />

r<br />

✲<br />

h<br />

❄<br />

Figure 15.25<br />

Let V be the volume and S be the surface area of the container. Then<br />

V = πr 2 h and S = 2πrh + 2πr 2


1072 Chapter Fifteen /SOLUTIONS<br />

where h is the height and r is the radius as shown in Figure 15.25. We have V = 100 cm 3 as our constraint. Since<br />

at the optimum<br />

∇S = (2πh + 4πr)⃗i + 2πr⃗j = π((2h + 4r)⃗i + 2r⃗j )<br />

and ∇V = 2πrh⃗i + πr 2 ⃗j = π(2rh⃗i + r 2 ⃗j ),<br />

∇S = λ∇V, we have<br />

π((2h + 4r)⃗i + 2r⃗j ) = πλ(2rh⃗i + r 2 ⃗j ),<br />

that is 2h + 4r = 2λrh and 2r = λr 2 , hence λ = 2 r .<br />

We assume r ≠ 0 or else we have a very awkward cylinder. Then, plug λ = 2/r into the first equation to obtain:<br />

( ) 2<br />

2h + 4r = 2 rh<br />

r<br />

Finally, solve for r and h using the constraint:<br />

2h + 4r = 4h<br />

h = 2r.<br />

V = πr 2 h = 100<br />

πr 2 (2r) = 100<br />

r 3 = 50<br />

π<br />

r = 3 √<br />

50<br />

π .<br />

√<br />

50<br />

Solving for h, we obtain h = 2r = 2 3 π .<br />

20. Constraint is G = P 1x + P 2y − K = 0.<br />

Since ∇Q = λ∇G, we have<br />

cax a−1 y b = λP 1 and cbx a y b−1 = λP 2.<br />

Dividing the two equations yields caxa−1 y b<br />

cbx a y = λP1 , or simplifying, ay<br />

b−1 λP 2 bx = P1<br />

. Hence, y = bP1 x.<br />

( ) P 2 aP 2<br />

bP 1 a + b<br />

Substitute into the constraint to obtain P 1x + P 2 x = P 1 x = K, giving<br />

aP 2 a<br />

x =<br />

aK<br />

bK<br />

and y = .<br />

(a + b)P 1 (a + b)P 2<br />

We now check that this is indeed the maximization point. Since x, y ≥ 0, possible maximization points are (0, K P 2<br />

),<br />

( K aK<br />

, 0), and ( ,<br />

P 1 (a + b)P 1<br />

aK<br />

that ( ,<br />

(a + b)P 1<br />

bK<br />

(a + b)P 2<br />

). Since Q = 0 for the first two points and Q is positive for the last point, it follows<br />

bK<br />

(a + b)P 2<br />

) gives the maximal value.<br />

21. (a) The company wishes to maximize P (x, y) given the constraint C(x, y) = 50, 000. The objective function is P (x, y)<br />

and the constraint equation is C(x, y) = 50, 000. The Lagrange multiplier λ is approximately equal to the change in<br />

P (x, y) given a one unit increase in the budget constraint. In other words, if we increase the budget by $1, we can<br />

produce about λ more units of the good.<br />

(b) The company wishes to minimize C(x, y) given the constraint equation P (x, y) = 2000. The objective function is<br />

C(x, y) and the constraint equation is P (x, y) = 2000. The Lagrange multiplier λ is approximately equal to the<br />

change in C(x, y) given a one unit increase in the production constraint. In other words, it costs about λ dollars to<br />

produce one more unit of the good.


15.3 SOLUTIONS 1073<br />

22. The company wants to maximize f(x, y) = 500x 0.6 y 0.3 given the constraint g(x, y) = 10x + 25y = 2000. Setting<br />

grad f = λ grad g gives<br />

500(0.6x −0.4 )y 0.3 = 10λ,<br />

500x 0.6 (0.3y −0.7 ) = 25λ.<br />

From the first equation we have λ = 30y 0.3 /x 0.4 , and from the second equation we have λ = 6x 0.6 /y 0.7 . Setting these<br />

equal gives<br />

y = 0.2x.<br />

Substituting this into the constraint equation 10x + 25y = 2000 gives x = 133.33. Since y = 0.2x, the maximum value<br />

occurs at x = 133.33 and y = 26.67.<br />

(a) The company should purchase 133.33 units of chemical X and 26.67 units of chemical Y. With these purchases, the<br />

company will be able to produce f(133.33, 26.67) = 25, 219 units of chemical Z.<br />

(b) When x = 133.33 and y = 26.67, we see that λ = 11.348. If $1 is added to the budget, the company will be able to<br />

produce about 11.348 additional units of chemical Z.<br />

23. (a) Let c be the cost of producing the product. Then c = 10W + 20K = 3000. At optimum production,<br />

∇q =<br />

(<br />

9<br />

2 W − 1 4 K<br />

1<br />

4<br />

)<br />

⃗ i +<br />

∇q = λ∇c.<br />

( )<br />

3<br />

W 3 4<br />

2 K<br />

− 3 4 ⃗ j , and ∇c = 10⃗i + 20⃗j . Equating we get<br />

9<br />

W − 1 1<br />

4<br />

2 K 4 = λ10, and<br />

3<br />

W 3 4<br />

2 K<br />

−<br />

4 3 = λ20.<br />

Dividing yields K = 1 W , so substituting into c gives<br />

6<br />

( ) 1<br />

10W + 20<br />

6 W = 40 3 W = 3000.<br />

Thus W = 225 and K = 37.5. Substituting both answers to find λ gives<br />

λ =<br />

9<br />

2 (225)− 1 4 (37.5)<br />

1<br />

4<br />

10<br />

= 0.2875.<br />

We also find the optimum quantity produced, q = 6(225) 4 3 1<br />

(37.5) 4 = 862.57.<br />

(b) At the optimum values found above, marginal productivity of labor is given by<br />

∣<br />

∂q<br />

∂W ∣ = 9 2 W − 1 1 ∣∣∣<br />

4 K 4 = 2.875,<br />

(225,37.5) (225,37.5)<br />

and marginal productivity of capital is given by<br />

∂q<br />

∣ = 3 ∂K<br />

2 W 3 4 K<br />

− 3 4 ∣<br />

∣<br />

(225,37.5)<br />

The ratio of marginal productivity of labor to that of capital is<br />

∂q<br />

∂W<br />

∂q<br />

∂K<br />

∣<br />

(225,37.5)<br />

= 1 2 = 10 cost of a unit of L<br />

=<br />

20 cost of a unit of K .<br />

= 5.750.<br />

(c) When the budget is increased by one dollar, we substitute the relation K 1 = 1 W1 into 10W1 + 20K1 = 3001 which<br />

6<br />

gives 10W 1 + 20( 1 40<br />

W1) = W1 = 3001. Solving yields W1 = 225.075 and K1 = 37.513, so q1 = 862.86 =<br />

6 3<br />

q + 0.29. Thus production has increased by 0.29 ≈ λ, the Lagrange multiplier.<br />

24. (a) The problem is to maximize<br />

subject to the budget constraint in dollars<br />

V = 1000D 0.6 N 0.3<br />

40000D + 10000N ≤ 600000<br />

or (in thousand dollars)<br />

40D + 10N ≤ 600


1074 Chapter Fifteen /SOLUTIONS<br />

(b) Let B = 40D + 10N = 600 (thousand dollars) be the budget constraint. At the optimum<br />

so<br />

Thus<br />

∇V = λ∇B,<br />

∂V<br />

∂D = λ ∂B<br />

∂D = 40λ<br />

∂V<br />

∂N = λ ∂B<br />

∂N = 10λ.<br />

∂V<br />

∂D<br />

∂V<br />

∂N<br />

Therefore, at the optimum point, the rate of increase in the number of visits to the number of doctors is four times the<br />

corresponding rate for nurses. This factor of four is the same as the ratio of the salaries.<br />

(c) Differentiating and setting ∇V = λ∇B yields<br />

Thus, we get<br />

So<br />

600D −0.4 N 0.3<br />

To solve for D and N, substitute in the budget constraint:<br />

40<br />

= 4.<br />

600D −0.4 N 0.3 = 40λ<br />

300D 0.6 N −0.7 = 10λ<br />

= λ = 300D0.6 N −0.7<br />

10<br />

N = 2D.<br />

600 − 40D − 10N = 0<br />

600 − 40D − 10 · (2D) = 0<br />

So D = 10 and N = 20.<br />

λ = 600(10−0.4 )(20 0.3 )<br />

≈ 14.67<br />

40<br />

Thus the clinic should hire 10 doctors and 20 nurses. With that staff, the clinic can provide<br />

V = 1000(10 0.6 )(20 0.3 ) ≈ 9,779 visits per year.<br />

(d) From part c), the Lagrange multiplier is λ = 14.67. At the optimum, the Lagrange multiplier tells us that about 14.67<br />

extra visits can be generated through an increase of $1,000 in the budget. (If we had written out the constraint in<br />

dollars instead of thousands of dollars, the Lagrange multiplier would tell us the number of extra visits per dollar.)<br />

(e) The marginal cost, MC, is the cost of an additional visit. Thus, at the optimum point, we need the reciprocal of the<br />

Lagrange multiplier:<br />

MC = 1 λ ≈ 1 ≈ 0.068 (thousand dollars)<br />

14.67<br />

i.e. at the optimum point, an extra visit costs the clinic 0.068 thousand dollars, or $68.<br />

This production function exhibits declining returns to scale (e.g. doubling both inputs less than doubles output,<br />

because the two exponents add up to less than one). This means that for large V , increasing V will require increasing<br />

D and N by more than when V is small. Thus the cost of an additional visit is greater for large V than for small. In<br />

other words, the marginal cost will rise with the number of visits.<br />

25. (a) The solution to Problem 23 gives λ = 0.29. We recalculate λ with a budget of $4000.<br />

The condition that grad q = λ grad(budget) in Problem 23 gives<br />

9<br />

2 W −1/4 K 1/4 = λ(10) and<br />

3<br />

2 W 3/4 K −3/4 = λ(20),<br />

so K = 1 W . Substituting into the budget constraint after replacing the budget of $3000 by $4000 gives<br />

6<br />

10W + 20( 1 6 W ) = 40 3 W = 4000.<br />

Thus, W = 300 and K = 50 and q = 1150.098.<br />

Multiplying the first equation by W and the second by K and adding gives<br />

W ( 9 2 W −1/4 K 1/4 ) + K( 3 2 W 3/4 K −3/4 ) = W (10λ) + K(20λ).


15.3 SOLUTIONS 1075<br />

So<br />

( 9<br />

2 + 3 2)<br />

W 3/4 K 1/4 = λ(10W + 20K)<br />

6W 3/4 K 1/4 = λ(4000)<br />

Thus,<br />

λ = 6W 3/4 K 1/4<br />

4000<br />

= 1150.098<br />

4000<br />

= 0.29<br />

Thus, the value of λ remains unchanged.<br />

(b) The solution to Problem 24 shows that λ = 14.67. We solve the problem again with a budget of $700,000.<br />

The condition that grad V = λ grad B in Problem 24 gives<br />

600D −0.4 N 0.3 = 40λ<br />

300D 0.6 N −0.7 = 10λ<br />

Thus, N = 2D. Substituting in the budget constraint after replacing the budget of 600 by 700 (the budget in measured<br />

in thousands of dollars) gives<br />

40D + 10(2D) = 700<br />

so D = 11.667 and N = 23.337 and V = 11234.705. As in part a), we multiply the first equation by D and the<br />

second by N and add:<br />

so<br />

D(600D −0.4 N 0.3 ) + N(300D 0.6 N −0.7 ) = D(40λ) + N(10λ),<br />

Since V = 1000D 0.6 N 0.3 = 11234.705, we have<br />

(600 + 300)D 0.6 N 0.3 = λ(400 + 10N)<br />

900D 0.6 N 0.3 = λ(700)<br />

λ = 900D0.6 N 0.3<br />

700<br />

= 9 7 ( V<br />

1000 ) = 14.44.<br />

Thus, the value of λ has changed with the budget.<br />

(c) We are interested in the marginal increase of production with budget (that is, the value of λ) and whether it is affected<br />

by the budget.<br />

Suppose $B is the budget. In part (a) we found<br />

and in part (b) we found<br />

λ = 6W 3/4 K 1/4<br />

B<br />

λ = 900D0.6 N 0.3<br />

.<br />

B<br />

In part (a), both W and K are proportional to B. Thus, W = c 1B and K = c 2B, so<br />

λ = 6(c1B)3/4 (c 2B) 1/4<br />

B<br />

= 6c3/4 1 C 1/4<br />

2 B 3/4 B 1/4<br />

B<br />

= 6c 3/4<br />

1 c 1/4<br />

2 .<br />

So we see λ is independent of B.<br />

In part (b), both D and N are proportional to B, so D = c 3B and N = c 4B. Thus,<br />

λ = 900(c3B)0.6 (c 4B) 0.3<br />

B<br />

= 900c0.6 3 C4 0.3 B 0.6 B 0.3<br />

B<br />

= 900c3 0.6 c4<br />

0.3 1<br />

B . 0.1


1076 Chapter Fifteen /SOLUTIONS<br />

So we see λ is not independent of B.<br />

The crucial difference is that the exponents in Problem 23 add to 1, that is 3/4+1/4 = 1, whereas the exponents<br />

in Problem 24 do not add to 1, since 0.6 + 0.3 = 0.9.<br />

Thus, the condition that must be satisfied by the Cobb-Douglas production function<br />

Q = cK a L b<br />

to ensure that the value of λ is not affected by production is that<br />

This is called constant returns to scale.<br />

26. (a) The curves are shown in Figure 15.26.<br />

a + b = 1.<br />

s<br />

1500<br />

I<br />

1000<br />

II<br />

III<br />

500<br />

(50, 500)<br />

s = 1000 − 10l<br />

20 40 60 80 100<br />

l<br />

Figure 15.26<br />

(b) The income equals $10/hour times the number of hours of work:<br />

s = 10(100 − l) = 1000 − 10l.<br />

(c) The graph of this constraint is the straight line in Figure 15.26.<br />

(d) For any given salary, curve III allows for the most leisure time, curve I the least. Similarly, for any amount of leisure<br />

time, curve III also has the greatest salary, and curve I the least. Thus, any point on curve III is preferable to any point<br />

on curve II, which is preferable to any point on curve I. We prefer to be on the outermost curve that our constraint<br />

allows. We want to choose the point on s = 1000 − 10l which is on the most preferable curve. Since all the curves<br />

are concave up, this occurs at the point where s = 1000 − 10l is tangent to curve II. So we choose l = 50, s = 500,<br />

and work 50 hours a week.<br />

27. The maximum of f(x, y) = ax 2 + bxy + cy 2 subject to the constraint g(x, y) = 1 where g(x, y) = x 2 + y 2 occurs<br />

where grad f = λ grad g. Since grad f = (2ax + by)⃗i + (bx + 2cy)⃗j and grad g = 2x⃗i + 2y⃗j we have<br />

2ax + by = 2xλ<br />

bx + 2cy = 2yλ<br />

x 2 + y 2 = 1<br />

Adding x times the first equation to y times the second gives x(2ax + by) + y(bx + 2cy) = (2x 2 + 2y 2 )λ. Dividing by<br />

2 and using the constraint equation gives f(x, y) = ax 2 + bxy + cy 2 = (x 2 + y 2 )λ = λ. This equation holds for all<br />

solutions (x, y, λ) of the three equations, including the solution that corresponds to the maximum value of f subject to<br />

the constraint. Thus the maximum value is f(x, y) = λ.<br />

28. (a) Let f(x 1, x 2, x 3) = ∑ 3<br />

i=1 xi2 = x 2 1 + x 2 2 + x 2 3 and g(x 1, x 2, x 3) = ∑ 3<br />

xi = 1. Then grad f = λ grad g<br />

i=1<br />

gives<br />

2x 1 = λ and 2x 2 = λ and 2x 3 = λ.<br />

so<br />

x 1 = x 2 = x 3 = λ 2 .


15.3 SOLUTIONS 1077<br />

Then x 1 + x 2 + x 3 = 1 gives<br />

3 λ 2 = 1 so λ = 2 3<br />

so x 1 = x 2 = x 3 = 1 3 .<br />

These values of x 1, x 2, x 3 give the minimum (rather than maximum) because the value of f increases without bound<br />

as x 2, x 2, x 3 → ∞.<br />

(b) A similar argument shows that ∑ n<br />

i=1 xi has its minimum value subject to ∑ n<br />

xi = 1 when<br />

i=1<br />

x 1 = x 2 = · · · = x n = 1 n .<br />

29. (a) The gradient vectors, ∇f, point inward around a local maximum. See the two points marked A in Figure 15.27.<br />

(b) Some of the gradient vectors around a saddle are pointing inward toward the point; some are pointing outward away<br />

from the point. See the point marked B in Figure 15.27.<br />

(c) The critical points on g = 1 are at points where ∇f is perpendicular to the curve g = 1. There are four of them, all<br />

marked with a dot in Figure 15.27. Imagine the level surfaces of f sketched in everywhere perpendicular to ∇f; the<br />

maximum value of f is at the point marked C in Figure 15.27<br />

(d) Again imagine level curves of f. The minimum value of f is at the point marked D.<br />

C<br />

A<br />

B<br />

A<br />

D<br />

Figure 15.27<br />

(e) At C, the maximum on g = 1, the vector ∇g points outward (because it points toward g = 2), while ∇f points<br />

inward. The Lagrange multiplier, λ, is defined so that ∇f = λ∇g, so λ must be negative.<br />

30. We want to minimize the function h(x, y) subject to the constraint that<br />

g(x, y) = x 2 + y 2 = 1,000 2 = 1,000,000.<br />

Using the method of Lagrange multipliers, we obtain the following system of equations:<br />

10x + 4y<br />

h x = −<br />

10,000 = 2λx,<br />

h y = −<br />

4x + 4y<br />

10,000 = 2λy,<br />

Multiplying the first equation by y and the second by x we get<br />

−y(10x + 4y)<br />

10,000<br />

x 2 + y 2 = 1,000,000.<br />

=<br />

−x(4x + 4y)<br />

.<br />

10,000


1078 Chapter Fifteen /SOLUTIONS<br />

Hence:<br />

2y 2 + 3xy − 2x 2 = (2y − x)(y + 2x) = 0,<br />

and so the climber either moves along the line x = 2y or y = −2x.<br />

We must now choose one of these lines and the direction along that line which will lead to the point of minimum<br />

height on the circle. To do this we find the points of intersection of these lines with the circle x 2 + y 2 = 1,000,000,<br />

compute the corresponding heights, and then select the minimum point.<br />

If x = 2y, the third equation gives<br />

5y 2 = 1,000 2 ,<br />

so that y = ±1,000/ √ 5 ≈ ±447.21 and x = ±894.43. The corresponding height is h(±894.43, ±447.21) = 2400 m.<br />

If y = −2x, we find that x = ±447.21 and y = ∓894.43. The corresponding height is h(±447.21, ∓894.43) =<br />

2900 m. Therefore, she should travel along the line x = 2y, in either of the two possible directions.<br />

31. The objective function is<br />

and the constraint is<br />

Partial derivatives of f and g are<br />

f(x, y, z) = √ (x − a) 2 + (y − b) 2 + (z − c) 2 ,<br />

g(x, y, z) = Ax + By + Cz + D = 0.<br />

f x =<br />

f y =<br />

f z =<br />

1 · 2 · (x − a)<br />

2<br />

= x − a<br />

f(x, y, z) f(x, y, z) ,<br />

1 · 2 · (y − b)<br />

2<br />

= y − b<br />

f(x, y, z) f(x, y, z) ,<br />

1 · 2 · (z − c)<br />

2<br />

= z − c<br />

f(x, y, z) f(x, y, z) ,<br />

Using Lagrange multipliers, we need to solve the equations<br />

g x = A, g y = B, and g z = C.<br />

grad f = λ grad g<br />

where grad f = f x<br />

⃗i + f y<br />

⃗j + f z<br />

⃗ k and grad g = gx ⃗i + g y<br />

⃗j + g z<br />

⃗ k . This gives a system of equations:<br />

Now x−a<br />

A<br />

= y−b<br />

B<br />

Substitute into the constraint,<br />

Hence<br />

x − a<br />

f(x, y, z) = λA<br />

y − b<br />

f(x, y, z) = λB<br />

z − c<br />

f(x, y, z) = λC<br />

Ax + By + Cz + D = 0.<br />

= z−c = λf(x, y, z) gives C<br />

x = A (y − b) + a,<br />

B<br />

z = C (y − b) + c,<br />

B<br />

( ) ( )<br />

A C<br />

A<br />

B (y − b) + a + By + C<br />

B (y − b) + c + D = 0,<br />

(<br />

A<br />

2<br />

B + B + C2<br />

B<br />

)<br />

y = A2<br />

B<br />

C2<br />

b − Aa + b − Cc − D.<br />

B


15.3 SOLUTIONS 1079<br />

Thus the minimum f(x, y, z) is<br />

y = (A2 + C 2 )b − B(Aa + Cc + D)<br />

,<br />

A 2 + B 2 + C 2<br />

−B(Aa + Bb + Cc + D)<br />

y − b =<br />

A 2 + B 2 + C 2<br />

x − a = A (y − b)<br />

B<br />

−A(Aa + Bb + Cc + D)<br />

=<br />

A 2 + B 2 + C 2<br />

z − c = C (y − b)<br />

B<br />

−C(Aa + Bb + Cc + D)<br />

=<br />

A 2 + B 2 + C 2<br />

√<br />

f(x, y, z) = (x − a) 2 + (y − b) 2 + (z − c) 2<br />

= [ ( ) 2 ( ) 2<br />

−A(Aa + Bb + Cc + D) −B(Aa + Bb + Cc + D)<br />

+<br />

A 2 + B 2 + C 2 A 2 + B 2 + C 2<br />

( ) 2<br />

−C(Aa + Bb + Cc + D) ] 1/2<br />

+<br />

A 2 + B 2 + C 2<br />

=<br />

|Aa + Bb + Cc + D|<br />

√<br />

A2 + B 2 + C 2 .<br />

The geometric meaning is finding the shortest distance from a point (a, b, c) to the plane Ax + By + Cz + D = 0.<br />

32. (a) The objective function is the energy loss, i 2 1R 1 + i 2 2R 2, and the constraint is i 1 + i 2 = I, where I is a constant. The<br />

Lagrangian function is<br />

L(i 1, i 2, λ) = i 2 1R 1 + i 2 2R 2 − λ(i 1 + i 2 − I).<br />

We look for solutions to the system of equations we get from grad L = ⃗0 :<br />

∂L<br />

= 2i 1R 1 − λ = 0<br />

∂i 1<br />

∂L<br />

= 2i 2R 2 − λ = 0<br />

∂i 2<br />

∂L<br />

= −(i1 + i2 − I) = 0.<br />

∂λ<br />

Combining ∂L − ∂L = 2(i 1R 1 − i 2R 2) = 0 with ∂L = 0 gives the two equation system<br />

∂i 1 ∂i 2 ∂λ<br />

i 1R 1 − i 2R 2 = 0<br />

i 1 + i 2 = I.<br />

Substituting i 2 = I − i 1 into the first equation leads to<br />

R 2<br />

i 1 = I<br />

R 1 + R 2<br />

R 1<br />

i 2 = I.<br />

R 1 + R 2<br />

(b) Ohm’s Law states that across a resistor<br />

Voltage = Current · Resistance.<br />

Since λ/2 = i 1 · R 1 = i 2 · R 2, the Lagrange multiplier λ equals twice the voltage across the resistors.


1080 Chapter Fifteen /SOLUTIONS<br />

33. (a) We draw the level curves (parallel straight lines) of f(x, y) = ax + by + c. We can see that the level lines with the<br />

maximum and minimum f-values which intersect with the disk are the level lines that are tangent to the boundary of<br />

the disk. Therefore, the maximum and minimum occur at the boundary of the disk. See Figure 15.28.<br />

f = max<br />

f = min<br />

▼<br />

f increases<br />

f = max<br />

Figure 15.28<br />

f = max<br />

f = min<br />

f increases❪<br />

f increases ✻<br />

f = min<br />

Figure 15.29<br />

Figure 15.30<br />

(b) Similar to part (a), we see the level lines with the largest and smallest f-values which intersect with the rectangle must<br />

pass the corner of the rectangle. So the maximum and minimum occur at the corners of rectangle. See Figure 15.29.<br />

When the level curves are parallel to a pair of the sides, then the points on the sides are all maximum or minimum, as<br />

shown below in Figure 15.30.<br />

(c) The graph of f is a plane. The part of the graph lying above a disk R is either a flat disk, in which case every point is<br />

a maximum, or is a tilted ellipse, in which case you can see that the maximum will be on the edge. Similarly, the part<br />

lying above a rectangle is either a rectangle or a tilted parallelogram, in which case the maximum will be at a corner.<br />

34. The point P is the solution to the constraint optimization problem of maximizing the square of the distance function.<br />

subject to the constraint<br />

D = x 2 + y 2 + x 2<br />

g(x, y, z) = f(x, y) − z = 0.<br />

(We take the square of the distance between the point (x, y, z) and the origin, which is<br />

Distance = √ (x − 0) 2 + (y − 0) 2 + (z − 0) 2 = √ x 2 + y 2 + z 2 ,<br />

because it makes the calculations easier.) Therefore, at point P , we have ∇D = λ∇g, so ∇D is parallel to ∇g.<br />

We know that ∇g is perpendicular to the surface g(x, y, z) = 0; that is, perpendicular to the surface z = f(x, y).<br />

Also<br />

∇D = 2x⃗i + 2y⃗j + 2z ⃗ k .


15.3 SOLUTIONS 1081<br />

At point P , whose position vector is ⃗p = a⃗i + b⃗j + c ⃗ k , we have<br />

∇D = 2(a⃗i + b⃗j + c ⃗ k ) = 2⃗p .<br />

Thus, ⃗p is parallel to ∇D and therefore ⃗p is also perpendicular to the surface.<br />

35. (a) The objective function f(x, y) = px + qy gives the cost to buy x units of input 1 at unit price p and y units of input<br />

2 at unit price q.<br />

The constraint g(x, y) = u tells us that we are only considering the cost of inputs x and y that can be used to<br />

produce quantity u of the product.<br />

Thus the number C(p, q, u) gives the minimum cost to the company of producing quantity u if the inputs it<br />

needs have unit prices p and q.<br />

(b) The Lagrangian function is<br />

L(x, y, λ) = px + qy − λ(xy − u).<br />

We look for solutions to the system of equations we get from grad L = ⃗0 :<br />

∂L<br />

∂x = p − λy = 0<br />

∂L<br />

∂y = q − λx = 0<br />

∂L<br />

= −(xy − u) = 0.<br />

∂λ<br />

We see that λ = p/y = q/x so y = px/q. Substituting for y in the constraint xy = u leads to x = √ qu/p,<br />

y = √ pu/q and λ = √ pq/u. The minimum cost is thus<br />

C(p, q, u) = p√ qu<br />

p + q √ pu<br />

q = 2√ pqu.<br />

36. (a) The objective function U(x, y) gives the utility to the consumer of x units of item 1 and y units of item 2.<br />

Since px + qy gives the cost to buy x units of item 1 at unit price p and y units of item 2 at unit price q, the<br />

constraint px + qy = I tells us that we are only considering the utility of inputs x and y that can be purchased with<br />

budget I.<br />

Thus the number V (p, q, I) gives the maximum utility the consumer can get with a budget of I if the two items<br />

have unit prices p and q.<br />

The indirect utility function tells how much utility the consumer can buy, depending on his budget and the prices<br />

of the two items.<br />

(b) The value of the Lagrange multiplier λ is the rate of change of the maximum utility V the consumer can get with<br />

his budget as the budget increases. This means that for small changes ∆I in the budget, smart buying will result in a<br />

change ∆V ≈ λ∆I in the utility to the consumer of his purchases.<br />

(c) The Lagrangian function is<br />

L(x, y, λ) = xy − λ(px + qy − I).<br />

We look for solutions to the system of equations we get from grad L = ⃗0 :<br />

∂L<br />

∂x = y − λp = 0<br />

∂L<br />

∂y = x − λq = 0<br />

∂L<br />

= −(px + qy − I) = 0.<br />

∂λ<br />

We see that λ = y/p = x/q so y = px/q. Substituting for y in the constraint px + qy = I leads to x = I/(2p),<br />

y = I/(2q) and λ = I/(2pq). The maximum utility is thus<br />

The marginal utility of money is<br />

V (p, q, I) = U( I 2p , I 2q ) = I 2p ·<br />

λ(p, q, I) = λ =<br />

I<br />

2pq .<br />

I<br />

2q = I2<br />

4pq .


1082 Chapter Fifteen /SOLUTIONS<br />

37. (a) The critical points of h(x, y) occur where<br />

h x(x, y) = 2x − 2λ = 0<br />

h y(x, y) = 2y − 4λ = 0.<br />

The only critical point is (x, y) = (λ, 2λ) and it gives a minimum value for h(x, y). That minimum value is m(λ) =<br />

h(λ, 2λ) = λ 2 + (2λ) 2 − λ(2λ + 4(2λ) − 15) = −5λ 2 + 15λ.<br />

(b) The maximum value of m(λ) = −5λ 2 + 15λ occurs at a critical point, where m ′ (λ) = −10λ + 15 = 0. At this<br />

point, λ = 1.5 and m(λ) = −5 · 1.5 2 + 15 · 1.5 = 11.25.<br />

(c) We want to minimize f(x, y) = x 2 + y 2 subject to the constraint g(x, y) = 15, where g(x, y) = 2x + 4y. The<br />

Lagrangian function is L(x, y, λ) = x 2 + y 2 − λ(2x + 4y − 15) so we solve the system of equations<br />

∂L<br />

= 2x − 2λ = 0<br />

∂x<br />

∂L<br />

= 2y − 4λ = 0<br />

∂y<br />

∂L<br />

= −(2x + 4y − 15) = 0.<br />

∂λ<br />

The first two equations give x = λ and y = 2λ. Substitution into the third equation gives 2λ + 4(2λ) − 15 = 0<br />

or λ = 1.5. Thus x = 1.5 and y = 3. The minimum value of f(x, y) subject to the constraint is f(1.5, 3) =<br />

1.5 3 + 3 2 = 11.25.<br />

(d) The two question have the same answer.<br />

38. You should try to anticipate your opponent’s choice. After you choose a value λ, your opponent will use calculus to<br />

find the point (x, y) that maximizes the function f(x, y) = 10 − x 2 − y 2 − 2x − λ(2x + 2y). At that point, we have<br />

f x = −2x − 2 − 2λ = 0 and f y = −2y − 2λ = 0, so your opponent will choose x = −1 − λ and y = −λ. This gives a<br />

value L(−1−λ, −λ, λ) = 10−(−1−λ) 2 −(−λ) 2 −2(−1−λ)−λ(2(−1−λ)+2(−λ)) = 11+2λ+2λ 2 which you<br />

want to make as small as possible. You should choose λ to minimize the function h(λ) = 11 + 2λ + 2λ 2 . You choose λ<br />

so that h ′ (λ) = 2 + 4λ = 0, or λ = −1/2. Your opponent then chooses (x, y) = (−1 − λ, −λ) = (−1/2, 1/2), giving<br />

a final score of L(−1/2, 1/2, −1/2) = 10.5. No choice of λ that you can make can force the value of L below 10.5. But<br />

your choice of λ = −1/2 makes it impossible for your opponent to force the value of L above 10.5.<br />

Solutions for Chapter 15 Review<br />

Exercises<br />

1. The critical points of f are obtained by solving f x = f y = 0, that is<br />

so<br />

f x(x, y) = 2y 2 − 2x = 0 and f y(x, y) = 4xy − 4y = 0,<br />

2(y 2 − x) = 0 and 4y(x − 1) = 0<br />

The second equation gives either y = 0 or x = 1. If y = 0 then x = 0 by the first equation, so (0, 0) is a critical point. If<br />

x = 1 then y 2 = 1 from which y = 1 or y = −1, so two further critical points are (1, −1), and (1, 1).<br />

Since<br />

D = f xxf yy − (f xy) 2 = (−2)(4x − 4) − (4y) 2 = 8 − 8x − 16y 2 ,<br />

we have<br />

D(0, 0) = 8 > 0, D(1, 1) = D(1, −1) = −16 < 0,<br />

and f xx = −2 < 0. Thus, (0, 0) is a local maximum; (1, 1) and (1, −1) are saddle points.<br />

2. At a critical point<br />

f x(x, y) = 2xy − 2y = 0<br />

f y(x, y) = x 2 + 4y − 2x = 0.


SOLUTIONS to Review Problems for Chapter Fifteen 1083<br />

From the first equation, 2y(x − 1) = 0, so either y = 0 or x = 1. If y = 0, then x 2 − 2x = 0, so x = 0 or x = 2. Thus<br />

(0, 0) and (2, 0) are critical points. If x = 1, then 1 2 + 4y − 2 = 0, so y = 1/4. Thus (1, 1/4) is a critical point. Now<br />

so<br />

D = f xxf yy − (f xy) 2 = 2y · 4 − (2x − 2) 2 = 8y − 4(x − 1) 2 ,<br />

D(0, 0) = −4, D(2, 0) = −4, D(1, 1 4 ) = 2<br />

so (0, 0) and (2, 0) are saddle points. Since f yy = 4 > 0, we see that (1, 1/4) is a local minimum.<br />

3. At a critical point<br />

f x(x, y) = 6x 2 − 6xy + 12x = 0<br />

f y(x, y) = −3x 2 − 12y = 0<br />

From the second equation, we conclude that −3(x 2 + 4y) = 0, so y = − 1 4 x2 . Substituting for y in the first equation<br />

gives<br />

6x 2 − 6x<br />

(− 1 )<br />

4 x2 + 12x = 0<br />

or<br />

x 2 + 1 4 x3 + 2x = x 4 (4x + x2 + 8) = 0.<br />

Thus x = 0 or x 2 + 4x + 8 = 0. The quadratic has no real solutions, so the only one critical point is (0, 0).<br />

At (0, 0), we have<br />

D(0, 0) = f xxf yy − (f xy) 2 = (12)(−12) − 0 2 = −144 < 0,<br />

so (0, 0) is a saddle point.<br />

4. The partial derivatives are<br />

Setting f x = 0 and f y = 0 gives<br />

f x = cos x + cos (x + y).<br />

f y = cos y + cos (x + y).<br />

cos x = cos y<br />

For 0 < x < π and 0 < y < π, cos x = cos y only if x = y. Then, setting f x = f y = 0:<br />

cos x + cos 2x = 0,<br />

cos x + 2 cos 2 x − 1 = 0,<br />

(2 cos x − 1)(cos x + 1) = 0.<br />

So cos x = 1/2 or cos x = −1, that is x = π/3 or x = π. For the given domain 0 < x < π, 0 < y < π, we only<br />

consider the solution when x = π/3 then y = x = π/3. Therefore, the critical point is ( π 3 , π 3 ).<br />

Since<br />

f xx(x, y) = − sin x − sin (x + y) f xx( π 3 , π 3 ) = − sin π 3 − sin 2π 3 = −√ 3<br />

f xy(x, y) = − sin (x + y) f xy( π 3 , π 3 ) = − sin 2π 3<br />

= − √ 3<br />

2<br />

f yy(x, y) = − sin y − sin (x + y) f yy( π 3 , π 3 ) = − sin π 3 − sin 2π 3 = −√ 3<br />

the discriminant is<br />

D(x, y) = f xxf yy − fxy<br />

2<br />

= (− √ 3)(− √ 3) − (− √ 3<br />

2 )2 = 9 > 0. 4<br />

Since f xx( π , π ) = −√ 3 < 0, ( π , π ) is a local maximum.<br />

3 3 3 3<br />

5. We find critical points:<br />

so (2, 3) is the only critical point. At this point<br />

f x(x, y) = 12 − 6x = 0<br />

f y(x, y) = 6 − 2y = 0<br />

D = f xxf yy − (f xy) 2 = (−6)(−2) = 12 > 0,<br />

and f xx < 0, so (2, 3) is a local maximum. Since this is a quadratic, the local maximum is a global maximum.<br />

Alternatively, we complete the square, giving<br />

f(x, y) = 10 − 3(x 2 − 4x) − (y 2 − 6y) = 31 − 3(x − 2) 2 − (y − 3) 2 .<br />

This expression for f shows that its maximum value (which is 31) occurs where x = 2, y = 3.


1084 Chapter Fifteen /SOLUTIONS<br />

6. The partial derivatives are f x = 2x − 3y, f y = 3y 2 − 3x. For critical points, solve f x = 0 and f y = 0 simultaneously.<br />

From 2x − 3y = 0 we get x = 3 2 y. Substituting it into 3y2 − 3x = 0, we have that<br />

3y 2 − 3( 3 2 y) = 3y2 − 9 2 y = y(3y − 9 2 ) = 0.<br />

So y = 0 or 3y − 9 2 = 0, that is, y = 0 or y = 3/2. Therefore the critical points are (0, 0) and ( 9 4 , 3 2 ).<br />

The contour diagram for f in Figure 15.31 (drawn by a computer), shows that (0, 0) is a saddle point and that ( 9 4 , 3 2 ) is a<br />

local minimum.<br />

y<br />

110<br />

4<br />

90 70<br />

50<br />

30<br />

10<br />

3<br />

2<br />

1<br />

0<br />

( 9 4 , 3 2 )<br />

−4 −3 −2 −1 1 2 3 4<br />

0<br />

−1<br />

10<br />

−10<br />

−2<br />

30<br />

0<br />

−30<br />

−3<br />

−50<br />

−70<br />

−90<br />

−4<br />

x<br />

Figure 15.31: Contour map of f(x, y) = x 2 + y 3 − 3xy<br />

We can also see that (0, 0) is a saddle point and ( 9 , 3 ) is a local minimum analytically. Since fxx = 2, fyy =<br />

4 2<br />

6y, f xy = −3, the discriminant is<br />

D(x, y) = f xxf yy − f 2 xy = 12y − (−3) 2 = 12y − 9.<br />

D(0, 0) = −9 < 0, so (0, 0) is a saddle point.<br />

D( 9 , 3 ) = 9 > 0 and 4 2 fxx = 2 > 0, we know that ( 9 , 3 ) is a local minimum. The point ( 9 , 3 ) is not a global minimum<br />

4 2 4 2<br />

since f( 9 , 3 ) = −1.6875, whereas f(0, −2) = −8.<br />

4 2<br />

7. The partial derivatives are<br />

f x = y + 1 , fy = x + 2y.<br />

x<br />

For critical points, solve f x = 0 and f y = 0 simultaneously. From f y = x + 2y = 0 we get that x = −2y. Substituting<br />

into f x = 0, we have<br />

y + 1 x = y − 1<br />

2y = 1 y (y2 − 1 2 ) = 0<br />

Since 1 ≠ 0, y y2 − 1 = 0, therefore 2<br />

y = ± √ 1<br />

√<br />

2<br />

= ±<br />

2 2 ,<br />

and x = ∓ √ (<br />

2. So the critical points are<br />

− √ 2, √ 2<br />

2<br />

)<br />

and<br />

( √2,<br />

−<br />

√<br />

2<br />

2<br />

)<br />

. But x must be greater than 0, so<br />

(<br />

− √ 2, √ 2<br />

2<br />

not in the domain.<br />

( √2, √ )<br />

The contour diagram for f in Figure 15.32 (drawn by computer), shows that − 2<br />

is a saddle point of f(x, y).<br />

2<br />

)<br />

is


SOLUTIONS to Review Problems for Chapter Fifteen 1085<br />

y<br />

f( √ √<br />

2, − 2<br />

2 ) = − 21<br />

2 + ln √ 2<br />

4<br />

3<br />

2<br />

1<br />

−1<br />

−2<br />

−3<br />

−4<br />

0<br />

−10<br />

❄<br />

✲<br />

✻<br />

−10<br />

0<br />

20<br />

30<br />

10<br />

4 5 6 7 8<br />

−20<br />

x<br />

Figure 15.32: Contour map of f(x, y) = xy + ln x + y 2 − 10<br />

( √2, √ )<br />

We can also see that − 2<br />

is a saddle point analytically.<br />

2<br />

Since f xx = − 1 , f<br />

x 2 yy = 2, f xy = 1, the discriminant is:<br />

√2, √ )<br />

D(<br />

−<br />

2<br />

2<br />

D(x, y) = f xxf yy − f 2 xy<br />

( √2, √ )<br />

= −2 < 0, so − 2<br />

is a saddle point.<br />

2<br />

= − 2 x 2 − 1.<br />

8. Note that the x-axis and the y-axis are not in the domain of f. Since x ≠ 0 and y ≠ 0, by setting f x = 0 and f y = 0 we<br />

get<br />

f x = 1 − 1 = 0 when x = ±1<br />

x2 f y = 1 − 4 = 0 when y = ±2<br />

y2 So the critical points are (1, 2), (−1, 2), (1, −2), (−1, −2). Since f xx = 2/x 3 and f yy = 8/y 3 and f xy = 0, the<br />

discriminant is<br />

( ) ( )<br />

D(x, y) = f xxf yy − fxy 2 2 8<br />

=<br />

− 0 2 = 16<br />

x 3 y 3 (xy) . 3<br />

Since D < 0 at the points (−1, 2) and (1, −2), these points are saddle points. Since D > 0 at (1, 2) and (−1, −2) and<br />

f xx(1, 2) > 0 and f xx(−1, −2) < 0, the point (1, 2) is a local minimum and the point (−1, −2) is a local maximum.<br />

No global maximum or minimum, since f(x, y) increases without bound if x and y increase in the first quadrant; f(x, y)<br />

decreases without bound if x and y decrease in the third quadrant.<br />

9. The objective function is f(x, y) = 3x − 4y and the constraint equation is g(x, y) = x 2 + y 2 = 5, so grad f = 3⃗i − 4⃗j<br />

and grad g = (2x)⃗i + (2y)⃗j . Setting grad f = λ grad g gives<br />

3 = λ(2x),<br />

−4 = λ(2y).<br />

From the first equation we have λ = 3/(2x), and from the second equation we have λ = −2/y. Setting these equal gives<br />

x = − 3 4 y.<br />

Substituting this into the constraint equation x 2 + y 2 = 5 gives y 2 = 16/5, so y = 4/ √ 5 and y = −4/ √ 5. Since<br />

x = − 3 y, there are two points where a maximum or a minimum might occur:<br />

4<br />

(−3/ √ 5, 4/ √ 5) and (3/ √ 5, −4/ √ 5).<br />

Since the constraint is closed and bounded, maximum and minimum values of f subject to the constraint exist. Since<br />

f(−3/ √ 5, 4/ √ 5) = −5 √ 5 and f(3/ √ 5, −4/ √ 5) = 5 √ 5, we see that f has a minimum value at (−3/ √ 5, 4/ √ 5) and<br />

a maximum value at (3/ √ 5, −4/ √ 5).


1086 Chapter Fifteen /SOLUTIONS<br />

10. The objective function is f(x, y) = x 2 + 2y 2 and the constraint equation is g(x, y) = 3x + 5y = 200, so grad f =<br />

(2x)⃗i + (4y)⃗j and grad g = 3⃗i + 5⃗j . Setting grad f = λ grad g gives<br />

2x = 3λ,<br />

4y = 5λ.<br />

From the first equation, we have λ = 2x/3, and from the second equation we have λ = 4y/5. Setting these equal gives<br />

x = 1.2y.<br />

Substituting this into the constraint equation 3x + 5y = 200 gives y = 23.256. Since x = 1.2y, we have x = 27.907. A<br />

maximum or minimum value of f can occur only at (27.907, 23.256).<br />

We have f(27.907, 23.256) = 1860.484. From Figure 15.33, we see that the point (27.907, 23.256) is a minimum<br />

value of f subject to the given constraint.<br />

50<br />

y<br />

40<br />

30<br />

20<br />

1000<br />

1860<br />

3000<br />

4000<br />

(27.9, 23.6)<br />

10<br />

3x + 5y = 200<br />

10 20 30 40 50<br />

x<br />

Figure 15.33<br />

11. The objective function is f(x, y) = 2xy and the constraint equation is g(x, y) = 5x + 4y = 100, so grad f =<br />

(2y)⃗i + (2x)⃗j and grad g = 5⃗i + 4⃗j . Setting grad f = λ grad g gives<br />

2y = 5λ,<br />

2x = 4λ.<br />

From the first equation we have λ = 2y/5, and from the second equation we have λ = x/2. Setting these equal gives<br />

y = 1.25x.<br />

Substituting this into the constraint equation 5x + 4y = 100 gives x = 10 and y = 12.5. A maximum or minimum value<br />

for f subject to the constraint can occur only at (10, 12.5).<br />

We have f(10, 12.5) = 250. From Figure 15.34, we see that the point (10, 12.5) gives a maximum.<br />

30<br />

y<br />

400<br />

20<br />

250<br />

10<br />

100<br />

(10, 12.5)<br />

5x + 4y = 100<br />

10 20<br />

x<br />

Figure 15.34


SOLUTIONS to Review Problems for Chapter Fifteen 1087<br />

12. We will use the Lagrange multipliers with:<br />

Objective function: f(x, y) = −3x 2 − 2y 2 + 20xy<br />

Constraint: g(x, y) = x + y − 100<br />

We first find<br />

To optimize f, we must solve the equations<br />

∇f = (−6x + 20y)⃗i + (−4y + 20x)⃗j<br />

∇g = ⃗i + ⃗j .<br />

We have a vector equation, so we equate the coordinates:<br />

∇f = λ∇g<br />

(−6x + 20y)⃗i + (−4y + 20x)⃗j = λ(⃗i + ⃗j ) = λ⃗i + λ⃗j<br />

So<br />

−6x + 20y = λ<br />

20x − 4y = λ.<br />

− 6x + 20y = 20x − 4y<br />

24y = 26x<br />

y = 13<br />

12 x<br />

Substituting into the constraint equation x + y = 100, we obtain:<br />

x + 13<br />

12 x = 100<br />

25<br />

12 x = 100<br />

x = 48.<br />

Consequently, y = 52, and f(48, 52) = 37,600. The point (48, 52) leads to the extreme value of f(x, y), given that<br />

x + y = 100. Note that f has no minimum on the line x + y = 100 since f(x, 100 − x) = −3x 2 − 2(100 − x) 2 +<br />

20x(100 − x) = −25x 2 + 2400x − 20000 which goes to −∞ as x goes to ±∞. Therefore, the point (48, 52) gives the<br />

maximum value for f on the line x + y = 100.<br />

13. Our objective function is f(x, y, z) = x 2 − y 2 − 2z and our equation of constraint is g(x, y, z) = x 2 + y 2 − z = 0.<br />

To optimize f(x, y, z) with Lagrange multipliers, we solve ∇f(x, y, z) = λ∇g(x, y, z) subject to g(x, y, z) = 0. The<br />

gradients of f and g are<br />

We get<br />

∇f(x, y, z) = 2x⃗i − 2y⃗j − 2 ⃗ k ,<br />

∇g(x, y, z) = 2x⃗i + 2y⃗j − ⃗ k .<br />

2x = 2λx<br />

−2y = 2λy<br />

−2 = −λ<br />

x 2 + y 2 = z.<br />

The third equation gives λ = 2 and from the first x = 0, from the second y = 0 and from the fourth z = 0. So the only<br />

solution is (0, 0, 0), and f(0, 0, 0) = 0.<br />

To see what kind of extreme point is (0, 0, 0), let (a, b, c) be a point which satisfies the constraint, i.e. a 2 + b 2 = c.<br />

Then f(a, b, c) = a 2 − b 2 − 2c = −a 2 − 3b 2 ≤ 0. The conclusion is that 0 is the maximum value of f and that there is<br />

no minimum.<br />

14. We first find the critical points in the disk<br />

∇z = (8x − y)⃗i + (8y − x)⃗j<br />

Setting ∇z = 0 gives 8x − y = 0 and 8y − x = 0. The only solution is x = y = 0. So (0, 0) is the only critical point in<br />

the disk.


1088 Chapter Fifteen /SOLUTIONS<br />

Next we find the extremal values on the boundary using Lagrange multipliers. We have objective function z =<br />

4x 2 − xy + 4y 2 and constraint G = x 2 + y 2 − 2 = 0.<br />

∇z = (8x − y)⃗i + (8y − x)⃗j<br />

∇G = 2x⃗i + 2y⃗j<br />

∇z = λ∇G gives<br />

If λ = 0 we get<br />

8x − y = 2λx<br />

8y − x = 2λy<br />

8x − y = 0<br />

8y − x = 0<br />

with only solutions x = y = 0, which does not satisfy the constraint: x 2 + y 2 − 2 = 0. Therefore λ ≠ 0 and we get:<br />

and<br />

2λy(8x − y) = 2λx(8y − x)<br />

y(8x − y) = x(8y − x).<br />

So x 2 = y 2 , x = ±y.<br />

Substitute into G = 0, we get 2x 2 − 2 = 0 so x = ±1. The extremal points on the boundary are therefore<br />

(1, 1), (1, −1), (−1, 1), (−1, −1). The region x 2 + y 2 ≤ 2 is closed and bounded, so minimum values of f in the region<br />

exist. We check the values of z at these points :<br />

z(1, 1) = 7, z(−1, −1) = 7, z(1, −1) = 9, z(−1, 1) = 9, z(0, 0) = 0<br />

Thus (−1, 1) and (1, −1) give the maxima over the closed disk and (0, 0) gives the minimum.<br />

15. The region x 2 ≥ y is the shaded region in Figure 15.35 which includes the parabola y = x 2 .<br />

y<br />

−70<br />

−50<br />

−30<br />

−10<br />

10<br />

30 50<br />

70<br />

x<br />

Figure 15.35<br />

of<br />

We first want to find the local maxima and minima of f in the interior of our region. So we need to find the extrema<br />

For this we compute the critical points:<br />

f(x, y) = x 2 − y 2 , in the region x 2 > y.<br />

f x = 2x = 0<br />

f y = −2y = 0.<br />

As (0, 0) does not belong to the region x 2 > y, we have no critical points. Now let’s find the local extrema of f on<br />

the boundary of our region, hence this time we have to solve a constraint problem. We want to find the extrema of<br />

f(x, y) = x 2 − y 2 subject to g(x, y) = x 2 − y = 0. We use Lagrange multipliers:<br />

grad f = λ grad g and x 2 = y.


SOLUTIONS to Review Problems for Chapter Fifteen 1089<br />

This gives<br />

2x = 2λx<br />

2y = λ<br />

x 2 = y.<br />

From the first equation we get x = 0 or λ = 1.<br />

If x = 0, from the third equation we get y = 0, so one solution is (0, 0). If x ≠ 0, then λ = 1 and from the second<br />

equation we get y = 1 . This gives 2 x2 = 1 so the solutions ( √<br />

1<br />

2 2<br />

, 1 ) and (− √<br />

1<br />

2 2<br />

, 1 ). 2<br />

So f(0, 0) = 0 and f( √ 1<br />

2<br />

, 1 ) = f(− √ 1<br />

2 2<br />

, 1 ) = 1 . From Figure 15.35 showing the level curves of f and the region<br />

2 4<br />

x 2 ≥ y, we see that (0, 0) is a local minimum of f on x 2 = y, but not a global minimum and that ( √ 1<br />

2<br />

, 1 ) and (− √<br />

1<br />

2 2<br />

, 1 ) 2<br />

are global maxima of f on x 2 = y but not global maxima of f on the whole region x 2 ≥ y.<br />

So there are no global extrema of f in the region x 2 ≥ y.<br />

16. Let the line be in the form y = b + mx. Then, when x equals 0, 1, and 2, y equals b, b + m, and b + 2m respectively.<br />

The sum of the squares of the vertical distances, which is what we want to minimize, is<br />

f(m, b) = (4 − b) 2 + (3 − (b + m)) 2 + (1 − (b + 2m)) 2<br />

To find critical points, set each partial derivative equal to zero.<br />

f m = 0 + 2(3 − (b + m))(−1) + 2(1 − (b + 2m))(−2)<br />

= 6b + 10m − 10<br />

f b = 2(4 − b)(−1) + 2(3 − (b + m))(−1) + 2(1 − (b + 2m))(−1)<br />

= 6b + 6m − 16<br />

Setting both partial derivatives equal to zero and dividing by 2, we get a system of equations:<br />

3b + 5m = 5<br />

3b + 3m = 8<br />

with solutions m = − 3 2 and b = 25 6 . Thus, the line is y = 25 6 − 3 2 x.<br />

Problems<br />

17. Since f xx < 0 and D = f xxf yy − f 2 xy > 0, the point (1, 3) is a maximum. See Figure 15.36.<br />

−120<br />

−64<br />

−32<br />

−16<br />

y<br />

0<br />

3<br />

−4<br />

−1<br />

✠<br />

1<br />

x<br />

Figure 15.36<br />

18. We first express the revenue R in terms of the prices p 1 and p 2:<br />

R(p 1, p 2) = p 1q 1 + p 2q 2<br />

= p 1(517 − 3.5p 1 + 0.8p 2) + p 2(770 − 4.4p 2 + 1.4p 1)<br />

= 517p 1 − 3.5p 2 1 + 770p 2 − 4.4p 2 2 + 2.2p 1p 2.


1090 Chapter Fifteen /SOLUTIONS<br />

At a local maximum we have grad R = 0, and so:<br />

Solving these equations, we find that<br />

∂R<br />

∂p 1<br />

= 517 − 7p 1 + 2.2p 2 = 0,<br />

∂R<br />

∂p 2<br />

= 770 − 8.8p 2 + 2.2p 1 = 0.<br />

p 1 = 110 and p 2 = 115.<br />

To see whether or not we have a found a local maximum, we compute the second-order partial derivatives:<br />

∂ 2 R<br />

∂p 2 1<br />

= −7,<br />

∂ 2 R<br />

∂p 2 2<br />

= −8.8,<br />

∂ 2 R<br />

∂p 1∂p 2<br />

= 2.2.<br />

Therefore,<br />

D = ∂2 R ∂ 2 R<br />

−<br />

∂2 R<br />

= (−7)(−8.8) − (2.2) 2 = 56.76,<br />

∂p 2 1 ∂p 2 2 ∂p 1∂p 2<br />

and so we have found a local maximum point. The graph of P (p 1, p 2) has the shape of an upside down paraboloid. Since<br />

P is quadratic in q 1 and q 2, (110, 115) is a global maximum point.<br />

19. Using Lagrange ( multipliers, ) ( let G = 2000)<br />

− 5x −(<br />

10y = 0 be ) the constraint. ( )<br />

∇P = 1 + 2xy2 ⃗i + 2 + 2yx2 ⃗j = 1 + xy2 ⃗i + 2 + yx2 ⃗j .<br />

2 · 10 8 2 · 10 8 10 8 10 8<br />

∇G = −5⃗i − 10⃗j .<br />

Now, ∇P = λ∇G, so<br />

Thus<br />

Solving, we get 2y = x or x = 0 or y = 0.<br />

1 + xy2<br />

yx2<br />

= −5λ and 2 +<br />

108 10 = −10λ.<br />

8<br />

2 + 2xy2<br />

10 8 = 2 + yx2<br />

10 8 .<br />

y<br />

200<br />

Constraint<br />

G = 0<br />

400<br />

x<br />

Figure 15.37<br />

From G = 0 we have: when x = 0, y = 200, when y = 0, x = 400, and when x = 2y, x = 200, y = 100. So<br />

(0,200), (400,0) and (200,100) are the critical points and they include the end points.<br />

Substitute into P : P (0, 200) = 400, P (400, 0) = 400, P (200, 100) = 402 so the organization should buy 200 sacks of<br />

rice and 100 sacks of beans.<br />

20. We want to minimize cost C = 100L + 200K subject to Q = 900L 1/2 K 2/3 = 36000. Using Lagrange multipliers, we<br />

get<br />

∇Q = ( 450L −1/2 K 2/3) ⃗i + ( 600L 1/2 K −1/3) ⃗j .<br />

∇C = λ∇Q gives<br />

Since λ ≠ 0 this gives<br />

∇C = 100⃗i + 200⃗j<br />

100 = λ450L −1/2 K 2/3 and 200 = λ600L 1/2 K −1/3 .<br />

450L −1/2 K 2/3 = 300L 1/2 K −1/3 .


Solving, we get L = (3/2)K. Substituting into Q = 36,000 gives<br />

SOLUTIONS to Review Problems for Chapter Fifteen 1091<br />

[<br />

Solving yields K =<br />

40 · ( 2<br />

3<br />

and L = 30 which gives C = $7, 000.<br />

21. We wish to minimize the objective function<br />

subject to the budget constraint<br />

900<br />

( 3<br />

2 K ) 1/2<br />

K 2/3 = 36,000.<br />

) ]<br />

1/2<br />

6/7<br />

≈ 19.85, so L ≈<br />

3<br />

(19.85) = 29.78. We can thus calculate cost using K = 20<br />

2<br />

C(x, y, z) = 20x + 10y + 5z<br />

Q(x, y, z) = 20x 1/2 y 1/4 z 2/5 = 1, 200.<br />

Therefore, we solve the equations grad C = λ grad Q and Q = 1, 200:<br />

20x 1/2 y 1/4 z 2/5 = 1, 200.<br />

The first and second equations imply that<br />

while the second and third equations imply that<br />

Substituting for x and z in the constraint equation gives<br />

20 = 10λx −1/2 y 1/4 z 2/5 or λ = 2x 1/2 y −1/4 z −2/5 ,<br />

10 = 5λx 1/2 y −3/4 z 2/5 , or λ = 2x −1/2 y 3/4 z −2/5 ,<br />

5 = 8λx 1/2 y 1/4 z −3/5 , or λ = 0.625x −1/2 y −1/4 z 3/5 ,<br />

x = y,<br />

3.2y = z.<br />

20y 1/2 y 1/4 (3.2y) 2/5 = 1200<br />

and so<br />

y ≈ 23.47,<br />

x ≈ 23.47 and z ≈ 75.1.<br />

22. We want to maximize f(x, y) = 80x 0.75 y 0.25 subject to the budget constraint g(x, y) = 6x + 4y = 8000. Setting<br />

grad f = λ grad g gives us<br />

80(0.75x −0.25 )y 0.25 = 6λ,<br />

80x 0.75 (0.25y −0.75 ) = 4λ.<br />

At the maximum, x, y ≠ 0. From the first equation we have λ = 10y 0.25 /x 0.25 , and from the second equation we have<br />

λ = 5x 0.75 /y 0.75 . Setting these equal gives<br />

y = 0.5x.<br />

Substituting this into the constraint equation 6x+4y = 8000, we see that x = 1000. Since y = 0.5x, we obtain y = 500.<br />

That the point (1000, 500) gives the maximum production is suggested by Figure 15.38, since the values of f decrease as<br />

we move along the constraint away from (1000, 500). To produce the maximum quantity, the company should use 1000<br />

units of labor and 500 units of capital.


1092 Chapter Fifteen /SOLUTIONS<br />

y<br />

2000<br />

Budget constraint: 6x + 4y = 8000<br />

1500<br />

1000<br />

500<br />

P = (1000, 500)<br />

f = 90000<br />

f = 80000<br />

50000<br />

f = 70000<br />

f = 60000<br />

500 1000 1500 x<br />

Figure 15.38<br />

23. (a) We want to minimize C subject to g = x + y = 39. Solving ∇C = λ∇g gives<br />

10x + 2y = λ<br />

2x + 6y = λ<br />

so y = 2x. Solving with x + y = 39 gives x = 13, y = 26, λ = 182. Therefore C = $4349.<br />

(b) Since λ = 182, increasing production by 1 will cause costs to increase by approximately $182. (because λ =<br />

‖∇ C‖<br />

= rate of change of C with g). Similarly, decreasing production by 1 will save approximately $182.<br />

‖∇ g‖<br />

24. (a) To be producing the maximum quantity Q under the cost constraint given, the firm should be using K and L values<br />

given by<br />

Hence 0.6aK−0.4 L 0.4<br />

0.4aK 0.6 L −0.6<br />

20K + 10<br />

∂Q<br />

∂K = 0.6aK−0.4 L 0.4 = 20λ<br />

∂Q<br />

∂L = 0.4aK0.6 L −0.6 = 10λ<br />

20K + 10L = 150.<br />

= 1.5 L<br />

K = 20λ<br />

10λ = 2, so L = 4 K. Substituting in 20K + 10L = 150, we obtain<br />

3<br />

( 4<br />

K = 150. Then K =<br />

3)<br />

9 2 and L = 6, so capital should be reduced by 1 unit, and labor should be<br />

2<br />

increased by 1 unit.<br />

(b)<br />

New production<br />

Old production = a4.50.6 6 0.4<br />

a5 0.6 5 0.4 ≈ 1.01, so tell the board of directors, “Reducing the quantity of capital by 1/2 unit<br />

and increasing the quantity of labor by 1 unit will increase production by 1% while holding costs to $150.”<br />

25. Cost of production, C, is given by C = p 1W + p 2K = b. At the optimal point, ∇q = λ∇C.<br />

Since ∇q = ( c(1 − a)W −a K a) ⃗i + ( caW 1−a K a−1) ⃗j and ∇C = p 1<br />

⃗i + p 2<br />

⃗j , we get<br />

Now, marginal productivity of labor is given by ∂q<br />

∂q<br />

∂K = caW 1−a K a−1 , so their ratio is given by<br />

c(1 − a)W −a K a = λp 1 and caW 1−a K a−1 = λp 2.<br />

∂q<br />

∂W<br />

∂q<br />

∂K<br />

∂W<br />

= c(1 − a)W −a K a and marginal productivity of capital is given by<br />

= c(1 − a)W −a K a<br />

caW 1−a K a−1<br />

= λp1<br />

λp 2<br />

= p1<br />

p 2<br />

which is the ratio of the cost of one unit of labor to the cost of one unit of capital.


SOLUTIONS to Review Problems for Chapter Fifteen 1093<br />

26. (a) Points A, B, C, D, E; that is, where a level curve of f and the constraint curve are parallel.<br />

(b) Point F since the value of f is greatest at this point.<br />

(c) Point D has the greatest f value of the points A, B, C, D, E.<br />

g = c<br />

❘<br />

A<br />

E<br />

D<br />

F<br />

13<br />

11<br />

12<br />

B<br />

10<br />

C<br />

9<br />

27. (a) By the method of Lagrange multipliers, the point (2, 1) is a candidate when the gradient for f at (2, 1) is a multiple of<br />

the gradient of the constraint function at (2, 1). The constraint function is g(x, y) = x 2 +y 2 , so grad g = 2x⃗i +2y⃗j .<br />

We have grad g(2, 1) = 4⃗i + 2⃗j . This is not a multiple of grad f(2, 1) = −3⃗i + 4⃗j , so (2, 1) is not a candidate.<br />

(b) The constraint function is g(x, y) = (x−5) 2 +(y+3) 2 , so grad g = 2(x−5)⃗i +2(y+3)⃗j . We have grad g(2, 1) =<br />

−6⃗i + 8⃗j . This is a multiple of grad f(2, 1) = −3⃗i + 4⃗j , so (2, 1) is a candidate.<br />

The contours near (2, 1) are parallel straight lines with increasing f-values as we move in the direction of<br />

−3⃗i + 4⃗j (approximately toward the northwest). The center of the constraint circle is at (5, −3), approximately<br />

southeast of the point (2, 1). Thus the point (2, 1) is a candidate for a maximum.<br />

In general, if the constraint is a circle and grad f points outside the circle, then the point is a candidate for a<br />

maximum.<br />

(c) The constraint function is g(x, y) = (x + 1) 2 + (y − 5) 2 . Thus grad g(2, 1) = 6⃗i − 8⃗j , which is again a multiple<br />

of grad f, so (2, 1) is a candidate.<br />

This time the center of the constraint circle, (−1, 5) is approximately northwest of (2, 1), the same general<br />

direction in which grad f is pointing. This means the point (2, 1) is a candidate for a minimum.<br />

In general, if the constraint is a circle and grad f points inside the circle, then the point is a candidate for a<br />

minimum.<br />

28. (a) (i) Suppose N = kA p . Then the rule of thumb tells us that if A is multiplied by 10, the value of N doubles. Thus<br />

2N = k(10A) p = k10 p A p .<br />

Thus, dividing by N = kA p , we have<br />

so taking logs to base 10 we have<br />

(where log 2 means log 10 2). Thus,<br />

(ii) Taking natural logs gives<br />

2 = 10 p<br />

p = log 2 = 0.3010.<br />

N = kA 0.3010 .<br />

ln N = ln(kA p )<br />

ln N = ln k + p ln A<br />

ln N ≈ ln k + 0.301 ln A<br />

Thus, ln N is a linear function of ln A.


1094 Chapter Fifteen /SOLUTIONS<br />

(b) Table 15.2 contains the natural logarithms of the data:<br />

Table 15.2<br />

ln N and ln A<br />

Island ln A ln N<br />

Redonda 1.1 1.6<br />

Saba 3.0 2.2<br />

Montserrat 2.3 2.7<br />

Puerto Rico 9.1 4.3<br />

Jamaica 9.3 4.2<br />

Hispaniola 11.2 4.8<br />

Cuba 11.6 4.8<br />

Using a least squares fit we find the line:<br />

This yields the power function:<br />

ln N = 1.20 + 0.32 ln A<br />

N = e 1.20 A 0.32 = 3.32A 0.32<br />

Since 0.32 is pretty close to log 2 ≈ 0.301, the answer does agree with the biological rule.<br />

29. (a) The objective function is the complementary energy,<br />

function is<br />

f 2 1<br />

2k 1<br />

+ f 2 2<br />

2k 2<br />

, and the constraint is f 1 +f 2 = mg. The Lagrangian<br />

L(f 1, f 2, λ) = f 2 1<br />

2k 1<br />

+ f 2 2<br />

2k 2<br />

− λ(f 1 + f 2 − mg).<br />

We look for solutions to the system of equations we get from grad L = ⃗0 :<br />

∂L<br />

∂f 1<br />

= f1<br />

− λ = 0<br />

k 1<br />

∂L<br />

∂f 2<br />

= f2<br />

− λ = 0<br />

k 2<br />

∂L<br />

= −(f1 + f2 − mg) = 0.<br />

∂λ<br />

Combining ∂L − ∂L = f1<br />

− f2<br />

= 0 with ∂L = 0 gives the two equation system<br />

∂f 1 ∂f 2 k 1 k 2 ∂λ<br />

f 1<br />

k 1<br />

− f2<br />

k 2<br />

= 0<br />

f 1 + f 2 = mg.<br />

Substituting f 2 = mg − f 1 into the first equation leads to<br />

f 1 =<br />

f 2 =<br />

k1<br />

k 1 + k 2<br />

mg<br />

k2<br />

k 1 + k 2<br />

mg.<br />

(b) Hooke’s Law states that for a spring<br />

Force of spring = Spring constant · Distance stretched or compressed from equilibrium.<br />

Since f 1 = k 1 · λ and f 2 = k 2 · λ, the Lagrange multiplier λ equals the distance the mass stretches the top spring<br />

and compresses the lower spring.


SOLUTIONS to Review Problems for Chapter Fifteen 1095<br />

√<br />

30. The distance to the origin d = x 2 + y 2 + z 2 . Minimizing d 2 minimizes d, so let d 2 = f(x, y, z) = x 2 + y 2 + z 2 . We<br />

minimize f with constraint z = xy + 1, so we let<br />

⎫<br />

g(x, y, z) = xy + 1 − z.<br />

2x = λy<br />

x = 0<br />

⎪⎬<br />

Lagrange: −→ ∇f = λ −→ 2y = λx<br />

y = 0<br />

∇g gives<br />

which gives<br />

2z = −λ<br />

z = 1<br />

⎪⎭<br />

z = xy + 1<br />

λ = −2.<br />

The point is (0, 0, 1). This must be a minimum since there are points on z = xy + 1 infinitely far from the origin.<br />

(There is no maximum.)<br />

31. Since patient 1 has a visit every x 1 weeks, this patient has 1/x 1 visits per week. Similarly, patient 2 has 1/x 2 visits per<br />

week. Thus, the constraint is<br />

g(x 1, x 2) = 1 + 1 = m<br />

x 1 x 2<br />

To minimize<br />

subject to g(x 1, x 2), we solve the equations<br />

This gives us the equations<br />

f(x 1, x 2) =<br />

Dividing the first equation by the second gives<br />

As v 1, v 2, x 1, x 2, m are strictly positive we have<br />

v1<br />

· x1<br />

v 1 + v 2 2 + v2<br />

· x2<br />

v 1 + v 2 2<br />

grad f = λ grad g<br />

g(x 1, x 2) = m.<br />

)<br />

∂f<br />

=<br />

v1<br />

· 1<br />

(−<br />

∂x 1 v 1 + v 2 2 = λ 1 x 2 1<br />

)<br />

∂f<br />

=<br />

∂x 2<br />

v2<br />

· 1<br />

(−<br />

v 1 + v 2 2 = λ 1 x 2 2<br />

1<br />

x 1<br />

+ 1 x 2<br />

= m.<br />

v 1<br />

= x2 2<br />

.<br />

v 2<br />

x 2 1<br />

= λ ∂g<br />

∂x 1<br />

= λ ∂g<br />

∂x 2<br />

Substituting for x 2 in the constraint occasion gives<br />

( ) 1<br />

x 2 v1 2<br />

= .<br />

x 1 v 2<br />

solving for x 1 gives<br />

( ) 1<br />

1 v2 2 1<br />

+ · = m<br />

x 1 v 1 x 1<br />

( )<br />

1<br />

1 + ( v1<br />

) 1 2 = m<br />

x 1 v 2<br />

x 1 = (v1) 1 2 + (v2) 1 2<br />

m · (v 1) 1 2<br />

,<br />

and similarly<br />

32. We want to optimize<br />

subject to<br />

f(x 1, x 2) =<br />

x 2 = (v1) 1 2 + (v2) 1 2<br />

m · (v 2) 1 2<br />

v1<br />

· x1<br />

v 1 + v 2 2 + v2<br />

· x2<br />

v 1 + v 2 2<br />

g(x 1, x 2) = 1 x 1<br />

+ 1 x 2<br />

= m.<br />

.


1096 Chapter Fifteen /SOLUTIONS<br />

At the optimum point, x 1, x 2, and the Lagrange multiplier λ must satisfy the equations<br />

v 1<br />

· 1<br />

v 1 + v 2 2 = − λ x 2 1<br />

v 2<br />

· 1<br />

v 1 + v 2 2 = − λ x 2 2<br />

1<br />

+ 1 = m.<br />

x 1 x 2<br />

Solving the first and second equations for 1/x 1 and 1/x 2, respectively, gives<br />

substituting into the constraint gives (note that λ < 0):<br />

So<br />

and thus<br />

(<br />

− 1<br />

2λ ·<br />

) 1/2 (<br />

v 1<br />

−1<br />

+<br />

(v 1 + v 2) 2λ ·<br />

1<br />

x 2 1<br />

1<br />

x 2 2<br />

v 1<br />

= − 1<br />

2λ · (v 1 + v 2)<br />

= − 1<br />

2λ · v 2<br />

(v 1 + v 2)<br />

) 1/2 (<br />

v 2<br />

= − 1 ) 1/2<br />

· (v1)1/2 + (v 2) 1/2<br />

= m.<br />

(v 1 + v 2)<br />

2λ (v 1 + v 2) 1/2<br />

− 1 v1 + v2 + 2(v1v2)1/2<br />

· = m 2 .<br />

2λ v 1 + v 2<br />

(<br />

)<br />

λ = − 1 1 + 2(v1v2)1/2<br />

.<br />

2m 2 v 1 + v 2<br />

The units of λ are weeks 2 (since the units of m are 1/weeks). The Lagrange multiplier measures df/dm, which<br />

represents the rate of change in the expected delay in tumor detection as the available number of visits per week increases.<br />

The negative sign represents the fact that as the number of visits per week increases, the delay decreases.<br />

33. We want to minimize the total cost, C, of the cable. The distance AB is √ 50 2 + x 2 meters and the distance BC is<br />

√<br />

302 + y 2 , and the cost for each segment is cost/meter × length. Thus<br />

where x, y, z must satisfy the constraint<br />

At the optimum point, ∇C = λ∇g, so<br />

√<br />

√<br />

C = 500 50 2 + x 2 + 2000 30 2 + y 2 + 300z,<br />

g(x, y, z) = x + y + z = 100.<br />

Thus, using λ = 300, we have<br />

∂C<br />

∂x =<br />

∂C<br />

∂y =<br />

∂C<br />

∂z<br />

500x<br />

√<br />

502 + x = λ = λ ∂g<br />

2 ∂x<br />

√ 2000y<br />

302 + y = λ = λ ∂g<br />

2 ∂y<br />

= 300 = λ = λ<br />

∂g<br />

∂z .<br />

2000y<br />

√<br />

302 + y 2 = 300<br />

20y = 3 √ 30 2 + y 2<br />

400y 2 = 9(900 + y 2 )<br />

391y 2 = 8100<br />

y = 90 √<br />

391<br />

= 4.6 meters.


Similarly<br />

500x<br />

√<br />

502 + x = 300<br />

2 √<br />

5x = 3 50 2 + x 2<br />

25x 2 = 9(2500 + x 2 )<br />

SOLUTIONS to Review Problems for Chapter Fifteen 1097<br />

16x 2 = 9 · 2500<br />

x = 3 · 50 = 37.5 meters.<br />

4<br />

Thus z = 100 − 37.5 − 4.6 = 57.9 meters. The values x = 37.5 m, y = 4.6 m, and z = 57.9 m give the minimum cost.<br />

How do we know these values of x, y, z give a minimum? Since the variables are constrained to lie in the first octant<br />

on the plane x + y + z = 100, we find the minimum cost on the boundary, which consists of three line segments on the<br />

coordinate planes, and compare with the cost at the critical point.<br />

√<br />

If x = 0, then z = 100 − y and the minimum of C(x, y, z) = 500 · 50 + 2000 30 2 + y 2 + 300(100 − y) for<br />

0 ≤ y ≤ 100 is C(0, 0, 100) = $115,000.<br />

If y = 0, then z = 100 − x and the minimum of C(x, y, z) = 500 √ 50 2 + x 2 + 2000 · 30 + 300(100 − x) for<br />

0 ≤ x ≤ 100 is C(37.5, 0, 62.5) = $110,000.<br />

If z = 0, then y = 100 − x and the minimum of C(x, y, z) = 500 √ √ 50 2 + x 2 + 2000 30 2 + (100 − x) 2 for<br />

0 ≤ x ≤ 100 is C(100, 0, 0) = $115,902.<br />

The cost at the critical point, C(37.5, 4.6, 57.9) = $109,321, is lower than the minimum $110, 000 on the boundary.<br />

Thus, the minimum cost subject to the constraint is C(37.5, 4.6, 57.9) = $109,321.<br />

34. The wetted perimeter of the trapezoid is given by the sum of the lengths of the three walls, so<br />

p = w +<br />

We want to minimize p subject to the constraint that the area is fixed at 50 m 2 . A trapezoid of height h and with parallel<br />

sides of lengths b 1 and b 2 has<br />

(b1 + b2)<br />

A = Area = h .<br />

2<br />

In this case, d corresponds to h and b 1 corresponds to w. The b 2 term corresponds to the width of the exposed surface of<br />

the canal. We find that b 2 = w + (2d)/(tan θ). Substituting into our original equation for the area along with the fact that<br />

the area is fixed at 50 m 2 , we arrive at the formula:<br />

Area = d (<br />

w + w +<br />

2d ) (<br />

= d w +<br />

d )<br />

= 50<br />

2<br />

tan θ<br />

tan θ<br />

2d<br />

sin θ<br />

We now solve the constraint equation for one of the variables; we will choose w to give<br />

Substituting into the expression for p gives<br />

We now take partial derivatives:<br />

p = w +<br />

w = 50 d −<br />

2d<br />

sin θ = 50<br />

d −<br />

d<br />

tan θ .<br />

d<br />

tan θ +<br />

2d<br />

sin θ .<br />

∂p<br />

∂d = − 50<br />

d − 1<br />

2 tan θ + 2<br />

sin θ<br />

∂p<br />

∂θ = d<br />

tan 2 θ · 1<br />

cos 2 θ − 2d<br />

sin 2 θ · cos θ<br />

From ∂p/∂θ = 0, we get<br />

d · cos 2 θ 1<br />

·<br />

sin 2 θ cos 2 θ = 2d · cos θ.<br />

sin 2 θ<br />

Since sin θ ≠ 0 and cos θ ≠ 0, canceling gives<br />

1 = 2 cos θ<br />

so<br />

cos θ = 1 2 .<br />

Since 0 < θ < π 2 , we get θ = π 3 .


1098 Chapter Fifteen /SOLUTIONS<br />

Substituting into the equation ∂p/∂d = 0 and solving for d gives:<br />

which leads to<br />

−50<br />

d 2 − 1 √<br />

3<br />

+ 2 √<br />

3/2<br />

= 0<br />

d =<br />

√<br />

50<br />

√<br />

3<br />

≈ 5.37m.<br />

Then<br />

w = 50<br />

d − d<br />

tan θ ≈ 50<br />

5.37 − 5.37 √ ≈ 6.21 m.<br />

3<br />

When θ = π/3, w ≈ 6.21 m and d ≈ 5.37 m, we have p ≈ 18.61 m.<br />

Since there is only one critical point, and since p increases without limit as d or θ shrink to zero, the critical point<br />

must give the global minimum for p.<br />

35. (a) A<br />

a<br />

A ′<br />

Medium 1<br />

θ 1<br />

R<br />

B ′<br />

✛<br />

Medium 2<br />

d<br />

Figure 15.39<br />

See Figure 15.39. The time to travel from A to B is given by<br />

T (θ 1, θ 2) = AR<br />

v 1<br />

(b) The distance d = A ′ B ′ = A ′ R + RB ′ . Hence<br />

+ RB<br />

v 2<br />

=<br />

θ 2<br />

✲<br />

b<br />

a<br />

v 1 cos θ 1<br />

+<br />

d = a tan θ 1 + b tan θ 2.<br />

B<br />

b<br />

v 2 cos θ 2<br />

.<br />

(c) We imagine the following extreme case: the light ray first travels through medium 1 to a point R on the boundary<br />

far to the left of A ′ , then through medium 2 toward B. The distance traveled this way is very large, hence the travel<br />

time is large as well. Similarly, if R is far to the right of B ′ , the travel time will be large. Therefore values of θ 1 near<br />

−π/2 or π/2 increase the time, T .<br />

(d) The constrained optimization problem is: minimize T (θ 1, θ 2) subject to g(θ 1, θ 2) = a tan θ 1 + b tan θ 2 = d.<br />

According to the method of Lagrange multipliers, the minimum point should be among those satisfying grad T =<br />

λ grad g as well as the constraint.We have<br />

grad T (θ 1, θ 2) = a v 1<br />

sin θ 1<br />

cos 2 θ 1<br />

⃗i + b<br />

v 2<br />

sin θ 2<br />

cos 2 θ 2<br />

⃗j<br />

and<br />

1 1<br />

grad g(θ 1, θ 2) = a ⃗i + b ⃗j .<br />

cos 2 θ 1 cos 2 θ 2<br />

The condition grad T = λ grad g becomes<br />

Eliminating λ we are left with<br />

a sin θ 1 1<br />

= λa<br />

v 1 cos 2 θ 1 cos 2 θ 1<br />

sin θ 1<br />

v 1<br />

and<br />

=<br />

b sin θ 2 1<br />

= λb .<br />

v 2 cos 2 θ 2 cos 2 θ 2<br />

or<br />

sin θ 1<br />

= v1<br />

,<br />

sin θ 2 v 2<br />

which is Snell’s law. The argument in part (c) shows that the critical point corresponding to θ 1, θ 2 satisfying Snell’s<br />

law is indeed a minimum.<br />

sin θ2<br />

v 2


SOLUTIONS to Review Problems for Chapter Fifteen 1099<br />

CAS Challenge Problems<br />

36. (a) The partial derivatives of f are<br />

∂f<br />

∂x =<br />

√ a + x +<br />

( −1 +<br />

√ a + x<br />

) y<br />

2 √ a + x √ a + x + y ( 1 + √ a + x + y ) 2<br />

∂f<br />

∂y = 1 − 2 a − 2 x + √ a + x − y<br />

2 √ a + x + y ( 1 + √ a + x + y ) 2<br />

Solving ∂f/∂x = 0, ∂f/∂y = 0, we get x = 1/4 − a, y = 1. The discriminant at this point is D = −16/625.<br />

Thus, by the second derivative test, the point is a saddle point.<br />

(b) The y coordinate of the critical points stays the same and the x coordinate is a<br />

√<br />

units to the left of its position when<br />

x + y<br />

a = 0. The type is always a saddle point. This is because f is obtained from<br />

1 + y + √ by substituting x + a for<br />

x<br />

x, so that the graph is shifted a units in the negative x-direction but its shape remains the same.<br />

37. (a) We have grad f = 2x⃗i + ⃗j and grad g = (2x + 2y)⃗i + (2x + 2y)⃗j . So the equations to be solved in the method<br />

of Lagrange multipliers are<br />

Solving these with a CAS, we get two solutions:<br />

x 2 + 2xy + y 2 − 9 = 0<br />

2x = λ(2x + 2y)<br />

1 = λ(2x + 2y)<br />

x = 1/2, y = −7/2, λ = −1/6, or x = 1/2, y = 5/2, λ = 1/6<br />

Student A reasons that since f(1/2, −7/2) = −13/4 and f(1/2, 5/2) = 11/4 , the (global) maximum and<br />

minimum values are 11/4 and 13/4, respectively. Student B graphs the constraint curve g = 0 and a contour diagram<br />

of f. The constraint curve turns out to be two straight lines, since the constraint x 2 + 2xy + y 2 − 9 = 0, which can<br />

be rewritten as (x + y) 2 = 9, or x + y = ±3. The value of f goes to infinity on each of these straight lines. On the<br />

line y = −x + 3, f(x, y) = x 2 + y = x 2 − x + 3, and on the line y = −x − 3, f(x, y) = x 2 + y = x 2 − x − 3.<br />

Thus Student B is correct. The points Student A found are actually local maximum and local minimum values, not<br />

global. Since the constraint is not bounded, there is no guarantee that there is a local maximum or minimum. See<br />

Figure 15.40.<br />

y<br />

4<br />

2<br />

−4 −2 2 4<br />

−2<br />

g<br />

x<br />

−4<br />

g<br />

Figure 15.40: Contours of f and two straight<br />

lines giving constraint g = 0<br />

38. (a) We have grad f = 3⃗i + 2⃗j and grad g = (4x − 4y)⃗i + (−4x + 10y)⃗j , so the Lagrange multiplier equations are<br />

3 = λ(4x − 4y)<br />

2 = λ(−4x + 10y)<br />

2x 2 − 4xy + 5y 2 = 20


1100 Chapter Fifteen /SOLUTIONS<br />

Solving these with a CAS we get λ = −0.4005, x = −3.9532, y = −2.0806 and λ = 0.4005, x = 3.9532, y =<br />

2.0806. We have f(−3.9532, −2.0806) = −11.0208, and f(3, 9532, 2.0806) = 21.0208. The constraint equation<br />

is 2x 2 − 4xy + 5y 2 = 20, or, completing the square, 2(x − y) 2 + 3y 2 = 20. This has the shape of a skewed ellipse,<br />

so the constraint curve is bounded, and therefore the local maximum is a global maximum. Thus the maximum value<br />

is 21.0208.<br />

(b) The maximum value on g = 20.5 is ≈ 21.0208 + 0.5(0.4005) = 21.2211. The maximum value on g = 20.2 is<br />

≈ 21.0208 + 0.2(0.4005) = 21.1008.<br />

(c) We use the same commands in the CAS from part (a), with 20 replaced by 20.5 and 20.2, and get the maximum<br />

values 21.2198 for g = 20.5 and 21.1007 for g = 20.2. These agree with the approximations we found in part (b) to<br />

2 decimal places.<br />

CHECK YOUR UNDERSTANDING<br />

1. True. By definition, a critical point is either where the gradient of f is zero or does not exist.<br />

2. False. The point P 0 could be a saddle point of f.<br />

3. False. The point P 0 could be a saddle point of f.<br />

4. True. If P 0 were not a critical point of f, then grad f(P 0) would point in the direction of maximum increase of f, which<br />

contradicts the fact that P 0 is a local maximum or minimum.<br />

5. True. The graph of this function is a cone that opens upward with its vertex at the origin.<br />

6. False. The graph of this function is a saddle shape, with a saddle point at the origin. The function increases in the ⃗i<br />

direction and decreases in the ⃗j direction.<br />

7. True. Adding 5 to the function shifts the graph 5 units vertically, which leaves the (x, y) coordinates of the local extrema<br />

intact.<br />

8. True. Multiplying by −1 turns the graph of f upside down, so local maxima become local minima and vice-versa.<br />

9. False. For example, the linear function f(x, y) = x + y has no local extrema at all.<br />

10. False. The statement is only true for points sufficiently close to P 0.<br />

11. False. Local maxima are only high points for f when compared to nearby values; the global maximum is the largest of<br />

any values of f over its entire domain.<br />

12. True. For unconstrained optimization, global extrema occur at one (or more) of the local extrema.<br />

13. False. For example, the linear function f(x, y) = x + y has neither a global minimum or global maximum on all of<br />

2-space.<br />

14. True. The region is the unit disk without its boundary (the unit circle), and the distance between any two points in this<br />

region is less than 2—it does not stretch off to infinity in any direction.<br />

15. False. The region is the unit disk without its boundary (the unit circle), so it is not closed (in fact, it is open).<br />

16. True. The global minimum is 0, which occurs at the origin. This is clear since the function f(x, y) = x 2 + y 2 is greater<br />

than or equal to zero everywhere, and is only zero at the origin.<br />

17. False. On the given region the function f is always less than one. By picking points closer and closer to the circle<br />

x 2 + y 2 = 1 we can make f larger and larger (although never larger than one). There is no point in the open disk that<br />

gives f its largest value.<br />

18. False. While f can only have (at most) one largest value, it may attain this value at more than one point. For example, the<br />

function f(x, y) = sin(x + y) has a global maximum of 1 at both (π/2, 0) and (0, π/2).<br />

19. True. The region is both closed and bounded, guaranteeing both a global maximum and minimum.<br />

20. True. The global minimum could occur on the boundary of the region.<br />

21. True. The point (a, b) must lie on the constraint g(x, y) = c, so g(a, b) = c.<br />

22. False. The point (a, b) is not necessarily a critical point of f, since it is a constrained extremum.<br />

23. True. The constraint is the same as x = y, so along the constraint f = 2x, which grows without bound as x → ∞.<br />

24. False. The condition grad f = λgrad g yields the two equations 1 = λ2x and 2 = λ2y. Substituting x = 2 in the first<br />

equation gives λ = 1/4, while setting y = −1 in the second gives λ = −1, so the point (2, −1) is not a local extremum<br />

of f constrained to x + 2y = 0.<br />

25. False. Since grad f and grad g point in opposite directions, they are parallel. Therefore (a, b) could be a local maximum<br />

or local minimum of f constrained to g = c. However the information given is not enough to determine that it is a


PROJECTS FOR <strong>CHAPTER</strong> <strong>FIFTEEN</strong> 1101<br />

minimum. If the contours of g near (a, b) increase in the opposite direction as the contours of f, then at a point with<br />

grad f(a, b) = λgrad g(a, b) we have λ ≤ 0, but this can be a local maximum or minimum.<br />

For example, f(x, y) = 4 − x 2 − y 2 has a local maximum at (1, 1) on the constraint g(x, y) = x + y = 2. Yet at<br />

this point, grad f = −2⃗i − 2⃗j and grad g = ⃗i + ⃗j , so grad f and grad g point in opposite directions.<br />

26. False. A maximum for f subject to a constraint need not be a critical point of f<br />

27. False. The condition for the Lagrange multiplier λ is grad f(a, b) = λ grad g(a, b).<br />

28. False. Just as a critical point need not be a maximum or minimum for unconstrained optimization, a point satisfying the<br />

Lagrange condition need not be a maximum or minimum for a constrained optimization.<br />

29. True. Since f(a, b) = M, we must satisfy the Lagrange conditions that f x(a, b) = λg(a, b) and f y(a, b) = λg y(a, b),<br />

for some λ. Thus f x(a, b)/f y(a, b) = g x(a, b)/g y(a, b).<br />

30. True. Since f(a, b) = m, the point (a, b) must satisfy the Lagrange condition that f x(a, b) = λg(a, b), for some λ. In<br />

particular, if g x(a, b) = 0, then f x(a, b) = 0.<br />

31. False. Whether increasing c will increase M depends on the sign of λ at a point (a, b) where f(a, b) = M<br />

32. True. The value of λ at a maximum point gives the proportional change in M for a change in c.<br />

33. False. The value of λ at a minimum point gives the proportional change in m for a change in c. If λ > 0 and the change<br />

in c is positive, the change in m will also be positive.<br />

PROJECTS FOR <strong>CHAPTER</strong> <strong>FIFTEEN</strong><br />

1. (a) If p = e −x where x → ∞ then p → 0 with p > 0 and<br />

lim (p ln p) = lim<br />

p→0<br />

x→∞ (−xe−x ) = 0,<br />

p>0<br />

since the exponential decreases faster than any power of x. Alternatively, use l’Hopital’s rule:<br />

lim<br />

p→0<br />

p>0<br />

ln p<br />

(p ln p) = lim<br />

p→0<br />

p>0<br />

1/p = lim<br />

p→0<br />

p>0<br />

1/p<br />

−1/p 2 = 0<br />

(b) We apply the method of Lagrange multipliers to find the critical points of S(p 1 , · · · , p 30 ). The constraint<br />

function is g(p 1 , · · · , p 30 ) = p 1 + · · · + p 30 . We have<br />

(<br />

∂S<br />

=<br />

∂ −<br />

∂p j ∂p j<br />

∑30<br />

i=1<br />

p i<br />

ln p i<br />

ln 2<br />

)<br />

= − 1<br />

ln 2 (ln p j + 1),<br />

therefore<br />

grad S = − 1<br />

ln 2<br />

∑30<br />

j=1<br />

(ln p j + 1) ⃗ k j<br />

where ⃗ k 1 , · · · , ⃗ k 30 are the unit vectors corresponding 30 independent directions of the p j -axes. Also,<br />

so the condition grad S = λ grad g becomes<br />

grad g =<br />

30∑<br />

j=1<br />

− 1<br />

ln 2 (ln p j + 1) = λ, for i = 1, · · · , 30.<br />

⃗k j<br />

Thus,<br />

ln p j = −λ ln 2 − 1


1102 Chapter Fifteen /SOLUTIONS<br />

and, in particular, all the p j s must be equal. Since the p j s have to satisfy the constraint g(p 1 , · · · , p 30 ) = 1,<br />

we see that p j = 1<br />

30 and that the point ( 1 30 , 1<br />

30 , · · · , 1<br />

30<br />

) is the only critical point of S. We have<br />

( 1<br />

S<br />

30 , 1<br />

30 , · · · , 1 )<br />

1 (− ln 30) ln 30<br />

= −30 ·<br />

=<br />

30 30 ln 2 ln 2 .<br />

We will not prove that this is indeed the maximum value of S (this requires a higher-dimensional analogue<br />

of the second derivative test). Since in part (c) we show that the minimum value of S is 0, the critical point<br />

we have found here is not a global minimum; the maximum of S has to be attained somewhere and it is<br />

reasonable to believe that it is attained at the unique critical point. The maximum entropy corresponds to<br />

maximum uncertainty in the outcome of the competition.<br />

(c) We already know that S ≥ 0. However, S can be zero: For example, if p 1 = 1 and p 2 = · · · = p 30 = 0,<br />

we have S(1, 0, · · · , 0) = 0. Therefore the minimum value of S is 0. Now we determine all the values of<br />

p i s for which S(p 1 , · · · , p 30 ) = 0. The condition<br />

2.<br />

∑30<br />

S = −<br />

i=1<br />

p i ln p i<br />

2<br />

together with the restrictions − ln p i ≥ 0 shows that, for S to vanish, each individual term in the above<br />

sum has to vanish. This means p i ln p i = 0 for all i = 1, · · · , 30, that is, p i = 0 or p i = 1 for i = 1, · · · , 30.<br />

Since ∑ 30<br />

i=1 p i = 1, only one of the p i s is 1 whereas the other 29 are 0. This corresponds to the case where<br />

one of the teams is certain to win, that is, there is no uncertainty. The result can be interpreted by saying<br />

that zero entropy implies zero uncertainty.<br />

= 0<br />

y = b + mx<br />

(x 1, y 1)<br />

(x i, b + mx i)<br />

(x 1, b + mx 1)<br />

(x i, y i)<br />

Figure 15.41<br />

(a) Points which are directly above or below each other share the same x coordinate, therefore, the point on<br />

the least squares line which is directly above or below the point in question will have x coordinate x i and<br />

from the formula for the least squares line, it will have y coordinate b + mx i . (See Figure 14.1.)<br />

(b) The general distance formula in two dimensions is d = √ (x 2 − x 1 ) 2 + (y 2 − y 1 ) 2 , so d 2 = (x 2 − x 1 ) 2 +<br />

(y 2 − y 1 ) 2 . Since the x coordinates are identical for the two points in question, the first term in the square<br />

root is zero. This yields d 2 = (y i − (b + mx i )) 2 .<br />

(c) In both cases we use the chain rule and our knowledge of summations to show the relationship.<br />

(<br />

∂f<br />

∂b = ∂ n<br />

)<br />

∑<br />

(y i − (b + mx i )) 2 =<br />

∂b<br />

i=1<br />

n∑<br />

= 2(y i − (b + mx i )) ·<br />

=<br />

i=1<br />

n∑<br />

2(y i − (b + mx i )) · (−1)<br />

i=1<br />

= −2<br />

n∑<br />

(y i − (b + mx i ))<br />

i=1<br />

n∑<br />

i=1<br />

∂<br />

∂b (y i − (b + mx i ))<br />

∂<br />

∂b (y i − (b + mx i )) 2


PROJECTS FOR <strong>CHAPTER</strong> <strong>FIFTEEN</strong> 1103<br />

(d) We can separate ∂f<br />

∂b<br />

∂f<br />

∂m =<br />

( ∂<br />

n<br />

)<br />

∑<br />

(y i − (b + mx i )) 2 =<br />

∂m<br />

i=1<br />

n∑<br />

= 2(y i − (b + mx i )) ·<br />

=<br />

i=1<br />

n∑<br />

2(y i − (b + mx i )) · (−x i )<br />

i=1<br />

= −2<br />

n∑<br />

(y i − (b + mx i )) · x i<br />

i=1<br />

into three sums as shown:<br />

i=1<br />

n∑<br />

i=1<br />

∂<br />

∂m (y i − (b + mx i ))<br />

( n<br />

)<br />

∂f<br />

∂b = −2 ∑ n∑ n∑<br />

y i − b 1 − m x i<br />

i=1<br />

Similarly we can separate ∂f<br />

∂m after multiplying through by x i:<br />

Setting ∂f<br />

∂b<br />

i=1<br />

i=1<br />

∂<br />

∂m (y i − (b + mx i )) 2<br />

i=1<br />

( n<br />

∂f<br />

∂m = −2 ∑<br />

n∑<br />

n∑<br />

y i x i − b x i − m<br />

∂f<br />

and equal to zero we have:<br />

∂m<br />

i=1<br />

bn + m<br />

n∑<br />

x i =<br />

i=1<br />

n∑<br />

n∑<br />

b x i + m x 2 i =<br />

i=1<br />

n∑<br />

i=1<br />

y i<br />

i=1<br />

n∑<br />

x i y i<br />

(e) To solve this pair of linear equations, we multiply the first equation by ∑ n<br />

by ∑ n<br />

i=1 x i, and subtract; we get<br />

n∑<br />

n∑<br />

bn x 2 i − b( x i ) 2 =<br />

i=1<br />

i=1<br />

n∑<br />

∑ n<br />

y i<br />

i=1 i=1<br />

i=1<br />

x 2 i −<br />

x i<br />

2<br />

n∑<br />

x i y i<br />

i=1<br />

)<br />

i=1 x2 i<br />

n<br />

∑<br />

i=1<br />

, multiply the second one<br />

x i ,<br />

So,<br />

Similarly,<br />

b =<br />

m =<br />

( n<br />

∑<br />

x 2 i<br />

i=1 i=1<br />

(<br />

n<br />

n∑<br />

y i −<br />

n∑<br />

x i y i −<br />

i=1<br />

n∑<br />

∑ n<br />

x i<br />

i=1 i=1<br />

n∑<br />

∑ n<br />

x i<br />

i=1 i=1<br />

) ⎛ (<br />

n∑ ∑ n<br />

x i y i / ⎝n x 2 i −<br />

i=1<br />

i=1<br />

) ⎛ (<br />

n∑ ∑ n<br />

y i / ⎝n x 2 i −<br />

i=1<br />

i=1<br />

x i<br />

) 2<br />

⎞<br />

⎠<br />

x i<br />

) 2<br />

⎞<br />

⎠<br />

(f) Applying the formulas to the given data, we have b = − 1 3<br />

, m = 1 which gives y = −(1/3) + x, in<br />

agreement with the example.

Hooray! Your file is uploaded and ready to be published.

Saved successfully!

Ooh no, something went wrong!